SlideShare uma empresa Scribd logo
1 de 75
VCE Physics
     Unit 4
     Topic 2


Interactions of
 Light & Matter
Unit Outline
To achieve the outcome the student should demonstrate the knowledge and skills to :
•   Explain the production of incoherent light from wide spectrum light sources including the
    Sun, light bulbs and candles (descriptive) in terms of thermal motion of electrons.
•   Explain the results of Young’s double slit experiment as evidence for the wave like nature
    of light including:
     –   constructive and destructive interference in terms of path difference
     –   qualitative effect of wavelength on interference patterns
•   Interpret the pattern produced by light when it passes through a gap or past an obstacle in
    terms of the diffraction of waves and the significance of the magnitude of the λ/w ratio
•   interpret the photoelectric effect as evidence of the particle like nature of light including the
    KE of emitted photoelectrons in terms of the energy of incident photons Ek max = hf – W, using
    energy units of both joules and electron-volts, effects of intensity of incident irradiation on
    the emission of photoelectrons.
•   interpret electron diffraction patterns as evidence of the wavelike nature of matter
    expressed as the De Broglie wavelength λ = h/p
•   compare momentum of photons and of particles of the same wavelength including
    calculations using p = h/λ
•   interpret atomic absorption and emission spectra, including those from metal vapour lamps
    in terms of the quantised energy level model of the atom, including calculations of the
    energy of photons emitted or absorbed. ∆E = hf
•   explain a model of quantised energy levels of the atom in which electrons are found in
    standing wave states
•   use safe and responsible practises when working with light sources, lasers and related
    equipment.
Chapter 1
Topics covered:
• The Nature of Light.
• Interference.
• Incoherent Light.
• Coherent Light
• Young’s Experiment.
• Path Difference
• Single Slit Diffraction
• Diffraction around Objects
1.0 The Nature of Light
                               EMR is a self propagating wave
Light is a form of ENERGY.     consisting of mutually
It is described as ELECTRO -   perpendicular, varying
MAGNETIC RADIATION (EMR).      ELECTRIC and MAGNETIC
                               FIELDS.
                               EMR travels through a vacuum
                               at 300,000 kms-1, (3.0 x 108 ms-1)
                                       Changing
                                      Magnetic Field
                                                         Direction of
                                                    Electromagnetic Wave
                                                          Movement
                                    Changing
                                   Electric Field
1.1 Superposition
                          Single waves, called         Once the
                          pulses. have the ability     superposition is
                          to pass through one          complete the pulses
                          another and, while           continue their
                          occupying the same           journey unaffected.
                          space, add together in a
                          process called                           A series of pulses
                          SUPERPOSITION.                           together form a
                                     Trough          Crest         wave train with
                                                                   alternating crests
When two wave trains interact                                      and troughs
with one another they also             Constructive interference occurs when
undergo SUPERPOSITION and              the two wave trains are in phase
will either:
1. Add together to produce a
larger wave - a process called
CONSTRUCTIVE INTERFERENCE
                 or
2. Subtract from one another to
                                       Destructive interference occurs when
produce no wave – a process called     the two wave trains are 1800 out of
DESTRUCTIVE INTERFERENCE               phase
1.2 Interference
                                                  Light behaves in a similar manner.
                                                  When light (with certain properties)
                               Destructive        is passed through two narrow slits,
                               Interference       an “interference pattern” is
                               Constructive       produced, showing constructive
                               Interference       (light bands) and destructive (dark
                                                  bands) interference.
                                                               Bright bands occur
                                                               where a crest and a
Waves, in this case water                                      crest (or a trough and a
waves, when passed                                             trough) arrive at the
through two narrow slits,                                      screen at the same time
“interfere” or interact with
one another to produce
areas of large disturbances                                         Dark bands
                                          Crest            Bright
(Constructive Interference)                                Band     occur when a
                                 Trough
or areas of no disturbance                                          crest and a
(Destructive Interference)                                          trough arrive at
                                                           Dark     the screen at
                                                           Band     the same time
1.3 Incoherent Light
Light is generated by luminous bodies, eg,
The Sun, light globes, burning candles.
Light is produced when atoms of the
filaments or source become electrically
excited and produce an electromagnetic
or light wave.                                Since the excitations occur in an
                                              unpredictably random fashion, the
Typical light sources such as those
                                              light waves are NOT produced in
mentioned above have an inbuilt
                                              regular repeating manner and so do
irregularity in the way they produce light.
                                              not maintain a constant “phase
                                              relationship” with one other.
                                              About once every 10-8 sec, a source
                                              will randomly alter its phase.
                                              Incoherent light, when combined,
This leads to the sources giving off a        produces rapidly moving areas of
broad spectrum of white light composed        constructive and destructive
of all colours in the rainbow.                interference and therefore do not
Each of the millions of colours have          produce a stable, visible interference
waves that are random to each other.          pattern.
This is called INCOHERENT LIGHT.
Light & Matter Revision
Question Type: Incandescent Light


  Q1: The light from a candle can best be described as
  A. coherent, arising from the vibrations of electrons.
  B. incoherent, arising only from the transition of electrons in excited
  energy levels falling to lower energy levels.
  C. coherent, arising only from the transition of electrons in excited energy
  levels falling to lower energy levels.
  D. incoherent, arising from the vibrations of electrons.
Light & Matter Revision
Question Type: Incandescent Light
 The spectrum of wavelengths produced by a particular
 incandescent light globe is shown in Figure 1 below.



                                                 Q2: The light produced by an
                                                 incandescent light globe can
                                                 best be described as
                                                 A. coherent.
                                                 B. incoherent.
                                                 C. monochromatic.
                                                 D. in phase.

  Q3: Describe the mechanism by which light is
  produced in an incandescent light globe.

  The thermal/random excitation of
  electrons in the filament leads to the
  emission of a broad/continuous spectrum.
1.4 Coherent Light
                                     Coherence is a property of waves that
                                     measures their ability to interfere with
  Coherent waves (zero phase         each other.
         difference)
Shown are monochromatic (single
colour) light waves of the same
frequency.                                     Coherent waves (constant
They are coherent and in phase, and will           phase difference)
combine constructively to produce bright
                                         Two waves that are coherent can
white light
                                         be combined to produce an
                                         unmoving distribution of
                                         constructive and destructive
                                         interference (a visible
                                         interference pattern) depending
                                         on the relative phase of the
                                         waves at their meeting point.
Lasers generate light at a single
wave length and frequency and all
of the waves (and PHOTONS) are in
phase.
This is called COHERENT LIGHT.
Light & Matter Revision
Question Type: Coherent Light


     Q4: Which of the following light sources will
     produce coherent, monochromatic light
     A. Sunlight
     B. LED
     C. Light globe with filter
     D. LASER
1.5 Young’s Experiment
                       First performed by      This pattern has a series of equally
  Thomas Young         Thomas Young in the     spaced coloured and black bands
  1773 –
  1829                 early 1800’s this       spread across the screen onto
                       experiment proved       which it is projected.
                       light was a wave.       The width of the coloured bands
                       It has been voted the   and their spacing depends on
                       most elegant
                      Incident Light
                                               the wavelength of the light used.
                       experiment ever         Short wavelength, BLUE light
                       devised.                produces a pattern with narrow blue
When light of a single frequency               bands which are closely spaced.
(colour) is passed through a pair
                                                    Long wavelength, RED light
of closely spaced, narrow slits
                                                    produces a pattern with wider
an “interference pattern” is
                                                    red bands which are spread
produced.
                                                    farther apart.


                                                                     BLUE Light
                                                                 using the same slits


                                                    Screen
                  Double Slits
                                                                      RED Light
Light & Matter Revision
Question Type: Young’s Experiment
 Thomas Young’s double slit experiment     The slits are now moved further
 has been replicated in the experimental   from the screen.
 arrangement shown in Figure 4.            Q6: What effect would this have
                                           on the pattern observed on the
                                           screen?
                                           The pattern will spread further
                                           across the screen.




  Q5: Explain using the wave theory of
  light why a series of bright and dark
  bars are observed on the screen.
 Slits provide 2 coherent sources
 leading to superposition which
 produces constructive and
 destructive interference leading to
 bright and dark bars on screen
Light & Matter Revision
Question Type: Young’s Experiment
  A physics teacher has
  apparatus to show
  Young’s double slit
  experiment. The
  apparatus is shown in
  Figure 4.
  The pattern of bright and
  dark bands is observed
  on the screen.




      Q7: Which one of the following actions will increase the distance, Δx,
      between dark bands in this double slit interference pattern?
      A. decrease the slit width
      B. decrease the slit separation
      C. decrease the slit screen distance
      D. decrease the wavelength of the light
1.6 Path Difference
                                 Light travels the same distance from S1 and S2
                                 to reach the central bright band, BC
                                 The path difference       The dark band (D1) on
         S1     S2               S1BC – S2BC = 0.          either side of AC occurs
                                 So a crest and a crest
                                                           because a crest and a
                                 (or trough and trough)
                                                           trough are arriving at the
                                 arrive at AC at the same
                                                           same time leading to
                                 time, leading to          destructive interference.
                                 constructive              Path difference:
                                 interference or a bright S1D1 – S2D1 = ½ λ
                                 band                     The next dark band (D2)
         Slits                   For the next bright
                                                          has path difference = 1½λ
         S1 S2                   band path difference
                                 S1B1 – S2B1 = 1 λ
                                 For the next bright       So dark bands occur
                                 band path difference      when
                                 would be:                 path difference = (n + ½)λ
                                  S1B2 – S2B2 = 2λ                   where n =
                                 So bright bands occur 0,1,2,3, etc
                                 when path difference = nλ
                                 where n = 0,1,2,3,etc
D2 B1   D1    B C D1   B1   D2
Light & Matter Revision
Question Type:   Interference – Path Difference
In the following diagram, laser light of
wavelength 600 nm is shone onto a pair of      Q9: Estimate the difference in
parallel slits and a pattern of alternating    length between P1 and P2.
light/dark bands is projected onto a wall.
                                                The indicated position is on
                                                the sixth antinode from the
                                                centre.
                                                The path difference is
                                                6λ = 6 × 600nm = 3.6 ×10−6 m



Q8: Explain how the observed pattern
on the wall supports a wave model for
light.
The alternating pattern represents an interference pattern which is a wave
phenomenon. The bright bands represent regions of constructive interference
where the difference in path length from each source is a multiple of
wavelengths. Waves therefore arrive from both sources in phase. The dark
regions represent bands of destructive interference with the waves being half
a wavelength out of phase due to path differences.
Light & Matter Revision
Question Type:   Interference – Path Difference
                                     Q10: What is the difference in length
                                     (S2P2 - S1P2) where P2 is the second
                                     maximum away from the central axis.


                                     Path difference to 2nd max = 2λ = 5.6 cm

Students have set up an experiment similar
to that of English physicist Thomas Young.
The students’ experiment uses microwaves
of wavelength λ = 2.8 cm instead of light.
The beam of microwaves passes through
two narrow slits shown as S1 and S2 in
Figure 3.
The students measure the intensity of the
resulting beam at points along the line
shown and determine the positions of
maximum intensity. These are shown as
filled circles and marked P0, P1 . . .
1.7 Single Slit Diffraction
                                                    Appreciable diffraction will occur if
When light of a single        THE EXTENT OF
                                                    the ratio λ/w is between about 0.1
wavelength shines             DIFFRACTION
                                                    and 50. Outside this range,
through a narrow gap or       DEPENDS UPON
                                                    diffraction is not observed.
single slit, a “diffraction   THE RATIO λ /w,
pattern” is produced.         WHERE λ =             Diffraction patterns for blue and
                              WAVELENGTH            red light show that the shorter
The pattern consists of       AND w = SLIT          wavelength blue light produces the
a rather wide, coloured       WIDTH.                more “compact” pattern, while




                                           Screen
central maximum with                                the longer wavelength red
a series of thinner                                 produces a more spread out
coloured and dark                                   pattern
bands spreading out                                          In other words, the spacing
from the centre.                                             between the lines is
                    Width = w                                wavelength dependent and
                    Single Slit                              patterns with the same line
Incident Light                                               spacing were made by light
Wavelength = λ                                               of the same wavelength
                                              BLUE light            using same slit




  Note: The wider the slit the narrower central maximum                RED light
Light & Matter Revision
Question Type: Single Slit Diffraction
In an experiment,
monochromatic laser light of
wavelength 600 nm shines
through a narrow slit, and the
intensity
of the transmitted light is
recorded on the screen some
distance away as shown in
Figure 2a. The intensity
pattern seen on the screen is
shown in Figure 2b.
Q11: Which one of the
intensity patterns (A-D) below
best indicates the pattern that
would be seen if a wider slit
was used?
1.8 Diffraction around Objects
In addition to diffraction                        Sunlight diffracting around
occurring when light passes                       a car tail light lens
through gaps, it can also
occur when light passes
around objects




                          The shadow of a
       Red Light          hand holding a coin
 Diffracting around a     illuminated by a He –
                                                               You can see a
       pinhead            Ne Laser.
                                                               diffraction
                              Point your hand                  pattern
                              toward a light                   through your
                              source and view                  fingers
                              through this gap
Chapter 2
Topics covered:
• Models for Light Behaviour.
• Particle like Properties of Light.
• Wave like Properties of Light.
2.0 Models for Light Behaviour
•   Light is/has energy and thus          Two separate models have
    can do work. We see this in            been proposed:
    things like plants growing or      1. The Particle Model:
    the chemical reactions on             This says that all properties of
    photographic films.                    light can be explained by
•   It is quite easy to list or show       assuming light is made up of
    what light DOES but more               a stream of individual
    difficult to say what light IS !       particles.
•   Much time and effort has
    been put into trying to find a     2. The Wave Model:
    THEORY or MODEL which
                                          This says that all properties of
    will explain ALL the
                                           light can be explained by
    properties and behaviours
                                           assuming light is made up of
    exhibited by light.
                                           a stream of waves.
2.1 Particle like Properties of
                   Light
•   Most properties of light can
    be explained using the “Wave
    Model”.
•   However, some properties
    can only be explained by
    assuming light is made up of
    a “stream of particles”.
                                     4. Certain colours of light can cause
•   These particle like properties   electrons to be emitted from some
    are:                             metal surfaces. This is called the
1. Light exerts pressure.            Photoelectric Effect.

2. Light can travel through a        5. Light reflects from shiny
    vacuum as well as through        surfaces (both a wave and particle like
    transparent media.               property).

                                     6. Different colours correspond to
3. Beams of light are bent by        different energies of light (both a wave
    gravity.                         and particle property).
2.2 Wave Like Properties of Light
•  Light exhibits many properties which
   can be demonstrated by other forms
   of waves eg. Water waves or waves on
   springs or strings.
• These wavelike properties of light are: 4. Light undergoes Diffraction when
                                              passing through narrow gaps.
1. Light travels very fast, but its speed
   does depend upon the medium            5. Light beams can pass through one
   through which it moves.                    another unaffected.

2. Light undergoes Reflection according   6. Different colours of light correspond
    to the Laws of Reflection.                to different energies.

3. Light undergoes Refraction according   7. Light forms interference patterns
    to the Laws of Refraction.                when passed through a pair of
                                              closely spaced, narrow slits.
Light & Matter Revision
Question Type: Light Properties

    Q12. Light sometimes behaves as a particle and
    sometimes as a wave. Which one or more of the
    following properties does light sometimes show?
    A. mass
    B. momentum
    C. charge
    D. energy
Light & Matter Revision
Question Type: Models for Light
Q 13: The table below contains some predictions for the
behaviour of light incident on a shiny metal sheet. Complete the
table by placing a .Y. (Yes) or .N. (No) in the appropriate boxes if
the prediction is supported by the wave and/or particle model of
light. Some answers have already been provided. It is possible for
predictions to be supported by both models


                                                        Y
                                                Y
                                                        N
                                                        N
Chapter 3
Topics covered:
• The Photoelectric Effect.
• Electron Energies
• Energy Units
• Practical Applications
3.0 The Photoelectric Effect
                                            Albert Einstein won his Nobel
The most important of the                   Prize for his explanation of this
particle like properties of                 phenomena not for his work on
light is the Photoelectric                  Relativity
Effect.
Simply put, the photoelectric
effect occurs when light
shines on a metal surface
causing electrons to be
ejected from the metal.
There are, however, certain restrictions:
(a) Only certain metals respond.
(b) Light must be above a certain
     frequency - the Cut Off Frequency.

                                             Einstein called these individual
                                             light particles PHOTONS


Louis De Broglie working in the 1920’s
suggested a photon looked like this.
3.1 The Photoelectric Effect (2)
The photoelectric effect is best studied                               Incident Photon
                                              Evacuated
using the demonstration circuit shown.        Glass Tube       e
An incident photon of the right frequency                             +
(that is above the Cut Off Frequency)
                                                 Ejected Electron
striking a susceptible metal (such as
aluminium, copper, lead, zinc and many                              Galvanometer   G
others), will free an electron.
                                                            Rheostat
The electron will cross the gap in the
evacuated tube, due to electrostatic
attraction.
                                                                          +


                                    So a stream of incident photons striking
                                    the metal will release a stream of electrons,
                                    setting up a current in the circuit which will
                                    be detected by the galvanometer (a
                                    sensitive ammeter able to detect currents
                                    in the μA range).
3.2 The Photoelectric Effect (3)
The current (of ejected photoelectrons)
does NOT depend upon the VOLTAGE
between the plates.
                                                                  Incident Photon
 The size of the current DOES depend       Evacuated
 upon the NUMBER of photons striking       Glass Tube        e
 the plate.                                                       +
The NUMBER of photons depends                  Ejected Electron
upon the INTENSITY of the incident
light.
Low Intensity or Dim Light of a
                                          Current (I)
particular colour or frequency will
produce a small current.                                              Bright Light
High Intensity or Bright Light of the
same frequency will produce a large                                   Dim Light
current.
                                                              Voltage (V)
3.3 The Photoelectric Effect (4)
To study the energy possessed by the
electrons ejected from the metal the           Evacuated
                                               Evacuated
                                               Evacuated
                                                Glass Tube +
                                                Glass Tube
                                               Glass Tube +
circuit is changed.                                        +
                                                             +
                                                             +
                                                             +
 The power supply has been reversed,                         +
 reversing the polarity of the plates.           Some e-e-electrons crossenough
                                                 No morenow don’t have the gap
                                                     All still have enough
                                                 thus energy to crossgap gap
                                                      Current = 0
                                                  energy to cross the the
                                                                   Galvanometer G
                                                                   Galvanometer
 Moving the slider on the rheostat
 increases the voltage between the
 plates.

This increasing voltage stops the least               Rheostat
                                                      Rheostat
                                                      Rheostat
energetic electrons crossing the gap,
reducing the current in the circuit.                             +
                                                                 +

The slider is again moved until no             Current (I)
electrons have enough energy to cross
the gap and the current drops to zero.

At this point the “Cut Off Voltage”
(VC) has been reached.
                                                                     Voltage (V)

                                          VC
3.4 The Photoelectric Effect (5)
                                    The fact that the current falls to zero
                                    only slowly as the reverse voltage
                                    increases indicates the photoelectrons
                                    are ejected from the metal surface with
                                    VARYING AMOUNTS OF ENERGY.

Different Intensities of the same               Current (I)
colour (frequency) have the
same cut off voltages


 Different frequencies and/or       -Vc                          Voltage (V)
 different metals will have
 differing cut off voltages
Light & Matter Revision
Question Type: Photoelectric effect
A student carries out an experiment to
                                          Q14: What is the maximum kinetic
investigate the photoelectric effect. She
                                          energy (in eV) of the electrons
shines a monochromatic light onto a
                                          ejected from the plate P?
metal plate (P) inside a sealed glass
                                          1.7 eV
chamber as shown in Figure 2.
                                          Q15: What is the subsequent
                                          maximum speed of these electrons
                                          ejected from plate P?
                                          KEMAX = 1.7 eV = (1.7)(1.6 x 10-19) J
                                                 = 2.72 x 10-19 J
                                          KE = ½ mv2
                                          v = 7.73 x 105 ms-1




 The current in the circuit changes as the voltage is varied as shown in Figure 3.
Light & Matter Revision
Question Type: Photoelectric effect
Blue light of wavelength 360 nm was        Q16: Determine the threshold
incident on a potassium metal plate. The   frequency of potassium.
photo electrons emitted from the            W = hfo
potassium plate were collected by a         fo = W/h
cathode and anode at varying voltages to       = 2.3/4.14 x 10-15
obtain the curve in Figure 3. Potassium        = 5.56 x 1014 Hz
has a work function of 2.3 eV.
                                                      Q18: Which of the
                                                      following (A-D) would
                                                      occur if the frequency
                                                      was decreased to less
                                                      than the threshold
                                                      frequency?
                                                      A Increased
                                                      photocurrent
                                                      B Decreased
                                                      photocurrent
Q17: UV light of 200 nm was now shone onto the        C Lower stopping
potassium plate at the same intensity striking the    potential
cathode. Sketch the resulting curve on the graph in   D No signal.
Figure 3 above.
3.5 Electron Energy
                                          When initially placed into the field, the
Before proceeding with more on            electron will possess Electrical Potential
the Photoelectric Effect a short          Energy given by the product of the
detour into the world of electron         charge on the electron (q) and the size of
energy is required.                       the voltage difference or accelerating
When an electron is placed in a           voltage (V) So, E.P.E. = qV
region where a voltage difference         At this point the electron is stationary
exists (ie. an electric field), it will   so its KE =0
undergo an energy conversion
in passing through that voltage
difference.

       The electron arrives at Plate 2                                     KE
                                                 KE = 0 KE   KE    KE
       where all its initial E.P.E. has          e-     e-   e-    e-      e-
       been converted to K.E. Thus               EPE EPE     EPE   EPE
         E.P.E.at start = K.E.at finish                                   EPE
                                                                          =0
       Mathematically:
                  qV = ½mv2
                                             Plate 1,                      Plate 2,
                                              V=0                          V = +V
3.6 Energy Units
•   Electrons are tiny and the amount of
    energy they carry, even when exposed
    to accelerating voltages of tens of                                       K
                                                    e-                        E
    millions of volts, is also tiny.                                              e-
•   In order to deal with this situation the
    normal energy unit of Joules (a very
    large unit) can be replaced by a                    EPE
    special energy unit called the
    “electron-Volt” (eV).
•                                            Plate 1, V = 0                  Plate 2, V = +1V
    The electron-Volt is defined as the
    energy change experienced by an
    electron in passing through a Voltage
                                                     On Plate 1 at Plate 2 thethe e-of
                                                      At arrival the E.P.E. of K.E.
    of 1 Volt.
                                                      = qVe- hasx 10-19 J = 1 eV eV
                                                      the = 1.6 increased by 1
•   Mathematically:
                   E.P.E. = qV                   Thus an electron passing through
                           = (1.6 x 10 )(1)
                                        -19
                                                 1 thousand volts will have
                           = 1.6 x 10-19 J       increased its energy by 1keV and
               Thus 1 eV = 1.6 x 10-19 J         an electron passing through 10
                                                 million volts will have increased its
                                                 energy by 10 MeV
3.7 The Photoelectric Effect (6)
•   The ENERGY of the incoming
    photons depends upon the              The Energy carried by the incoming
    FREQUENCY OF THE LIGHT. So,           Photon is TOTALLY ABSORBED by
    blue photons are more energetic       the electron with which it collides.
    than red ones.
•   Mathematically:                       Some of the photon energy is used free
                  E = hf                  the electron from the metal lattice and the
    where E = Energy (J or eV)            rest is converted into the Kinetic Energy
           h = Planck’s Constant          of the ejected electron. This can be
           f = Frequency (Hz)
                                          summarised mathematically as:
•   Planck’s constant can have 2 values
    depending upon the energy units
                                                   KEMAX = hf - W
    used:
    If Joules:    h = 6.63 x 10-34 J s;    where:
    if eV:        h = 4.14 x 10-15 eVs    KEMAX = Maximum KE of              ejected
                                          electron (J or eV)                 hf =
                                          Photon Energy (J or eV)            W=
                                          Work Function (J or eV)

                                          The term, W, called the Work Function, is
                                          the energy needed to bring the electron
                                          from within the metal lattice to the surface
                                          before it can be ejected.
3.8 The Photoelectric Effect (7)
A plot of the KEMAX   1. Remember KE MAX = hf - W,                  3. Different metals have
of the ejected        thus a graph of KE MAX vs                     different cut off
electron versus the   frequency is a straight line                  frequencies.
frequency of the      graph of the form y = mx + c
                                                                    4. Different metals have
incoming photons is   with m = h and c = W
                                                                    different work functions.
shown below. This     2. When KEMAX = 0, hf = W
graph has a number    and this frequency is called                  5. All metals will produce
of important          the “cut off frequency” (fO)                  graphs with the same
features:                                                           slope = h (Planck’s
                      for that particular metal.
                                                                    constant)
                          Light below this frequency,
                          no matter how intense, will never
                             liberate electrons from this
                 K.E.MAX                 metal.
                 (eV or J )
                                                        Slope = h        Slope = h



                              fo                  fo
                                                                     Frequency (Hz)

            W

            W
Light & Matter Revision
Question Type: Photoelectric effect
Susan and Peter conducted a photo-
electric experiment in which they used a
light source and various filters
to allow light of different frequencies to fall
on the metal plate of a photo-electric cell.
The maximum kinetic energy of any emitted
photo-electrons was determined by
                                                  Figure 3 shows the stopping
measuring the voltage required, VS
                                                  voltage, VS, as a function of the
(stopping voltage), to just stop them
reaching the collector electrode. The             frequency (f) of the light falling on
apparatus is shown in Figure 2.                   the plate.

 Table 1 shows the work
 functions for a series of metals.
Light & Matter Revision
Question Type: Photoelectric effect
Q19: Use the information above to
identify the metal surface used in
Susan and Peter’s experiment.
The work function corresponding to
sodium could be found by drawing a
line of best fit through the points on the
graph to determine the y-intercept.

Q20: Use the results in Figure 3 to
calculate the value for Planck’s constant
that Susan and Peter would have
obtained from the data.
You must show your working.


The gradient of the line of best fit gave Planck’s
constant as approximately 4.5 x 10-15 eV s.
Light & Matter Revision
Question Type: Photoelectric effect
Measurements of the kinetic energy of
electrons emitted from potassium
metal were made at a number of
frequencies. The results are shown in
Figure 1.
Q21: What is the minimum energy
of a light photon that can eject an
electron from potassium metal?
 1.8 eV
The work function for silver
metal is higher than the work
function for potassium metal.

Q22: Which one of the graphs (A-
D) would best describe the result
if the experiment was repeated
with silver metal instead of
potassium metal?
Light & Matter Revision
Question Type: Photoelectric effect
Some students are
investigating the
photoelectric effect. They
shine light of different
wavelengths onto a
rubidium plate. They
measure the maximum
kinetic energy of
photoelectrons emitted
from the plate. Their data
of maximum kinetic energy     Q23: From the data on the graph, what is the
of ejected photoelectrons     minimum energy, W, required to remove
as a function of the          photoelectrons from the rubidium plate?
frequency of incident light   2.0 eV
is shown in Figure 1.         The students shine light of wavelength λ = 400 nm
In answering the following    onto the rubidium plate.
questions, you must use       Q24: From the graph, with what maximum kinetic
the data from the graph.      energy would the photoelectrons be emitted?
Take the speed of light to
                              1.0 eV
be 3.0 × 108 m s-1
3.9 The Photoelectric Effect (8)
   PHOTOELECTRIC EFFECTS OBSERVED                    WAVE MODEL PREDICTIONS

1. Below a certain frequency,               1. The energy of the light beam
different for different metals, NO          arrives uniformly and continuously at
photoelectrons are observed, no             the metal surface. If the intensity
matter how INTENSE the light.               increases, the energy available to the
                                            electrons increases, so KEMAX of the
                                            ejected electrons should also
                                            increase.
2. The KEmax of the ejected                2. The energy of the light beam is
photoelectrons is INDEPENDENT              completely described by its INTENSITY,
of the INTENSITY of the incident           so the KEMAX of the ejected
light, but DEPENDENT on the                photoelectrons should be INDEPENDENT
FREQUENCY of that light.                   of Frequency.

                                            3. Because of the nature of the energy
3. The photoelectrons are emitted
                                            delivery process, electrons should take
IMMEDIATELY the metal is exposed to
                                            SOME TIME to build enough energy to
light above the cut off frequency.
                                            be ejected. A TIME DELAY should exist.

These anomalies between the observed and predicted results meant the death of the wave
theory as a complete explanation for light behaviour. A new theory was required.
Light & Matter Revision
Question Type: Photoelectric effect


 The students use a light source that     Q25: Comment whether this
 emits a large range of frequencies.      experimental evidence supports
 They use filters which allow only        the wave like or the particle-like
 certain frequencies from the source      theory of light.
 to shine onto the plate. Most of the
 students filters produce frequencies    The experimental evidence supported
 below the cut-off frequency.            the particle theory.
 Alice says that if they increase the
                                         The wave theory predicts that
 intensity of light, these frequencies
                                         photoelectrons would be emitted at
 below the cut-off
                                         any frequency if the intensity was
 frequency will be able to produce
                                         sufficient.
 emitted photoelectrons.
 They experiment and find Alice is        the particle theory, the energy of the
 incorrect.                              photons is related to the frequency,
                                         not the intensity, so there would be
                                         no electrons emitted below the
                                         threshold frequency.
3.10 The Photoelectric Effect (9)
As mentioned previously it      Photons are neither          Photons can be
was in 1905 that Einstein       waves nor particles,         visualised as a series of
proposed that light travels     having properties            individual particles each
as a series of discrete units   similar to particles when    of which displays some
or particles or “quanta”        travelling through a         wave like properties.
which he called “photons”       vacuum and when in a
and each carried a discrete     gravitational field, while
amount of energy                also having properties
dependent upon the light’s      similar to waves when
                                                                Individual Photon
frequency. (ie E = hf).         refracting and
                                interfering.

                   Light beam


                                                              Light Beams can be
                                                              regarded as a series of
                                                              photons radiating out
                                                              from the source.
3.11 Practical Applications
The photoelectric effect has
practical applications in many
areas:
  Solar Cells

                                 Spacecraft
                                 The photoelectric effect will cause
                                 spacecraft exposed to sunlight to
                                 develop a positive charge.
                                 This can get up to the tens of volts.
                                 This can be a major problem, as
 Night Vision Goggles            other parts of the spacecraft in
                                 shadow develop a negative charge
                                 (up to several kilovolts) from
                                 nearby plasma, and the imbalance
                                 can discharge through delicate
                                 electrical components.
Chapter 4
Topics covered:
• Investigating the Electron
• Electrons and Matter as Waves.
• Electron Interference
• Electron Diffraction
• Electrons and X Rays
• Photon Momentum.
• Photons as Waves.
4.0 Investigating the Electron
                                            •   Electrons are stripped from atoms
                                                and then used as individual
                                                particles in many applications.
                                            •   One important application is in
                                                Cathode Ray Tubes - the basis for
                                                Cathode Ray Oscilloscopes
•   The electron is the smallest of the 3       (CRO’s), T.V. and non LCD video
    fundamental particles which make            display units.
    up all atoms.                           •   These devices rely on a stream of
•   It carries the basic unit of electric       “energetic” electrons, which are
    charge (1.6 x 10-19 C)                      “boiled off” a hot wire filament.
•   It has a mass of 9.1 x 10-31 kg         •   Their energy is then increased by
•   When in an atom, the electron               being accelerated through a large
    circulates around the nucleus BUT           Voltage.
    only in certain allowed positions or    •   Their trajectory can be manipulated
    orbits.                                     by using magnetic or electric fields.
•   This restriction on position means
    that electrons are behaving more like
    waves than the particles we know
    them to be.
•   It was not until the development of
    “Quantum Mechanics” that the
    problem of restricted position was
    satisfactorily explained.
4.1 Electrons and Matter as Waves
            In 1924 French Physicist
            Louis DeBroglie suggested
            that “the universe is
  symmetrical” so, if radiation (light)     A racing car, mass 800 kg, travelling at
  displayed both particle and wave          200 kmh-1 (55.6 ms-1) has a DeBroglie
  like properties simultaneously,           wavelength of:
  matter should also display similar                    λ = h/p = h/mv
                                                         = 6.63 x 10-34/(800)(55.6)
  behaviours.
                                                           = 1.5 x 10-38 m
• DeBroglie suggested that matter (with     An indescribably small number. Too
   mass, m, moving with velocity, v), will  small to measure or even notice.
   have a wavelength (called the DeBroglie
   wavelength) associated with its motion.
                                                                        v
• This wavelength is calculated from the
   momentum equation:                      An electron of mass 9.1 x 10-31 kg travelling
                λ = h/p = h/mv             at 1.4 x 104 ms-1 has a DeBroglie wavelength
where λ = DeBroglie wavelength (m)                                 of:
                   h = Planck’s Constant                   λ = h/p = h/mv
                   p = Particle Momentum         = 6.63 x 10-34/(9.1 x 10-31)(1.4 x 104)
   (kgms )
         -1
                   m = Mass (kg)                            = 5.2 x 10-8 m
                   v = Velocity (ms )
                                   -1
                                           A measurable quantity, so electrons are
                                              able to display wave like properties like
                                              interference and diffraction.
Light & Matter Revision
Question Type: De Broglie Wavelength
A sketch of a cathode ray tube (CRT) is       Q26: Calculate the de Broglie
shown in Figure 5. In this device, electrons wavelength of the electrons. You
of mass 9.10 × 10-31 kg are                   must show your working.
accelerated to a velocity of 2.0 × 10 m s . A
                                     7    -1
                                               λ= h/mv
fine wire mesh in which the gap between the
wires is w = 0.50 mm                           the de Broglie wavelength was
has been placed in the path of the electrons, found to be 3.64 x 10-11 m.
and the pattern produced is observed on the Q27: Explain, with reasons,
fluorescent screen.                            whether or not the students
                                               would observe an electron
                                               diffraction pattern on the
                                               fluorescent screen due to the
                                               presence of the mesh.
                                              For diffraction, the gap width
                                              must be the same order of
                                              magnitude as the wavelength.
                                              Since the wavelength was much
                                              smaller than the gap, no
                                              diffraction pattern would be
                                              observed.
Light & Matter Revision
Question Type: De Broglie Wavelength
Neutrons are subatomic particles and, like electrons, can
exhibit both particle-like and wave-like behaviour.
A nuclear reactor can be used to produce a beam of
neutrons, which can then be used in experiments.
The neutron has a mass of 1.67 × 10-27 kg.
The neutrons have a de Broglie wavelength of 2.0 × 10-10 m.

 Q28: Calculate the speed of the neutrons.
  λ= h/mv gives
  2.0 × 103 ms–1 (or 1985 ms–1)

Q29: The neutron beam is projected onto a
metal crystal with interatomic spacing of 3.0 ×
10-10 m.
Would you expect to observe a diffraction
pattern? Explain your answer.
There would be a diffraction pattern because
the wavelength is of the same order of
magnitude as the interatomic spacing.
3.11 Practical Applications
The photoelectric effect has
practical applications in many
areas:
  Solar Cells

                                 Spacecraft
                                 The photoelectric effect will cause
                                 spacecraft exposed to sunlight to
                                 develop a positive charge.
                                 This can get up to the tens of volts.
                                 This can be a major problem, as
 Night Vision Goggles            other parts of the spacecraft in
                                 shadow develop a negative charge
                                 (up to several kilovolts) from
                                 nearby plasma, and the imbalance
                                 can discharge through delicate
                                 electrical components.
4.2 Electron Interference
                 The now familiar Young’s       Initially, with only small
   .
       . .. .    double slit experiment is      numbers, electrons arrive
                 performed with the light       at the screen in a random
                 source replaced by an          fashion.
                 “electron gun” which fires     Even as the numbers reach
                 a single electron at a time.   the thousands still nothing
                                                unusual appears on the
                                                screen.
                                                However as the numbers
                                                climb past 5000 the familiar
                                                light and dark bands begin
                                                to appear
                                                The total exposure time
8 electrons                  270 electrons      from picture (a) to the
                                                picture (d) was 20 min.

                                                This result shows
                                                particles (in this case
                                                electrons) can display
                                                wave like properties.
2000 electrons               6000. electrons
4.3 Electron Diffraction
                                 Just as waves diffract
                                 when hitting a solid
                                 object so electrons
                                 can diffract from a
                                 well-ordered
                                 arrangement of atoms
                                 on the surface of a
                                 sample.

                                                Electrons which have been
                                                accelerated through a potential
                                                of 30 to 500 volts (i.e., have
The arrangement of the                          K.E.’s of 30 to 500 eV), have a
spots is interpreted to    Platinum Sample      de Broglie wavelength between
provide information about                       2.2 x 10-10 m and 0.5 x 10-10 m.
the ordered arrangement of                      This fits nicely into the range of
atoms on the surface and                        distances between atoms in
the distances between the                       solids and can therefore
spots gives information on                      strongly diffract from them.
the distance between the
atoms.                      e- energy = 65 eV
Light & Matter Revision
Question Type: Electron Diffraction
 A beam of electrons, pass through two
 very thin slits (approximately 10-10 m)
                                           Q30: Explain whether the
 and are detected on a electron detector
                                           particle model or the wave
 screen as shown in Figure 5.
                                           model best explains the
                                           expected observations from
                                           this experiment.

                                           The wave model best explains
                                           this.
                                           The wave model is better:
                                           Interference pattern (a wave
                                           phenomenon) will be
                                           observed.
                                           The Particle model would not
                                           predict the interference
                                           pattern, rather two zones
                                           where the electrons would
                                           strike the screen.
4.4 Electrons and X Rays
                  Diffraction
                   Pattern


       Al foil



High energy
 X Ray Beam
electron beam




A high energy electron
beam is fired at an             The distance between the bright lines in
Aluminium target                both patterns is the same, meaning the
The diffraction pattern         wavelengths of both beams was the same.
shown is produced.
                                Hence if λX Ray is known, then the De
When the electron beam is       Broglie wavelength of the electrons is
replaced with an X Ray          also known.
beam a somewhat similar
diffraction is produced
Light & Matter Revision
Question Type: X ray Energy
A beam of X-rays, wavelength λ = 250
pm (250 × 10-12 m), is directed onto a
thin aluminium foil as shown in Figure
4a. The X-rays scatter from the foil onto
the photographic film.

                                               Q31 : Calculate the energy, in
                                               keV, of these X-rays.




                                       E=hc/λ
                                        = (4.14 x 10-15)(3.0 x 108)/ 250 x 10-12)
                                        = 4.97 x 103 ev
                                        = 4.97 keV or 5.0 keV
Light & Matter Revision
Question Type: X ray Diffraction
  After the X-rays pass through the foil,    Q32: Explain why the electrons
  a diffraction pattern is formed as         produce a diffraction pattern similar
  shown in Figure 4b.                        to that of the X-rays.
  In a later experiment, the X-rays are     Electrons have a de Broglie
  replaced with a beam of energetic         wavelength which was the same
  electrons. Again, a diffraction pattern   (or very similar) to the wavelength
  is observed which is very similar to      of the X-rays.
  the X-ray diffraction pattern. This is    Q33: Assuming the two diffraction
  shown in Figure 4c.                       patterns are identical, estimate the
                                            momentum of the electrons. Include
                                            the unit.
                                            p=h/λ
                                              = (6.63 x 10-34)/(250 x 10-12)
                                              = 2.7 x 10-24 kgms-1
                                            The diffraction pattern were the
                                            same, so the wavelength of the
                                            electrons must equal that of the X-
                                            rays.
Light & Matter Revision
Question Type: X ray & electron Diffraction
 In 1927, G.I. Thomson fired a beam of
 electrons through a very thin metal foil
 and the emerging electrons formed
 circular patterns, similar to the pattern
 obtained when the exercise was repeated
 with a beam of X-rays. The following
 diagram shows a comparison of the
 patterns obtained.
                                           Q35: If the X-rays have frequency of
Q34: How does this evidence support the 1.5 x 1019 Hz, what is the wavelength
concept of matter waves?                   of the electrons?
The circular pattern is a diffraction
pattern. With X-rays, the bright lines       λ = c/f = 3.0 x 108/1.5 x 1019
represent regions of constructive              = 2.0 x 10-11 m
interference alternating with regions of Q36: What is the momentum of an X-
destructive interference.                  ray photon?
The corresponding electron pattern
shows similar diffraction properties         p = h/λ = 6.63 x 10-34/2.0 x 10-11
which suggests electrons have wave             = 3.3 x 10-23 kgms-1
properties, as diffraction is a wave
phenomenon.
4.5 Evidence for Photons as Waves
                                         It appears trying to predict the fate of
                                         individual photons is not possible,
Young’s double slit experiment is
                                         but as their numbers build the wave
well known as evidence for the wave
                                         model allows the prediction of their
like nature of light.
                                         average behaviour with great
This experiment creates difficulties
                                         accuracy.
for the particle (photon) model.
                                         Thus photons (en masse) can display
How can individual photons, which
                                         wave like properties although they
must pass through one or other of
                                         are generally regarded as discrete,
the slits, interact with themselves to
                                         individual particles.
produce an interference pattern ?
When the experiment is carried out
at extremely low light intensities, it
is found that, for a while, no
interference pattern is noticed.
However, as time passes the
number of photons arriving at the
screen builds and an interference
pattern does emerge with more
photons going to the areas of
bright bands and less to areas of
dark bands.
4.6 Photon Momentum
•   When a light beam strikes a surface, the
    individual photons will transfer MOMENTUM
    as they are absorbed or reflected and so will
    exert a pressure on that surface.
•   The size of the MOMENTUM can be
    calculated from
                  p = E/c …………(1)
    where, p = Photon Momentum (kgms-1)
           E = Photon Energy (J or eV)
           c = Speed of Light (ms-1)

•   Using Photon Energy, E = hf, Equation 1 can
    be rewritten as:
         p = hf/c …………(2)
•   Using the Wave Equation, c = fλ
•   Equation 2 can be rewritten as:
                  p = hf/f λ
                  p = h/λ
Light & Matter Revision
Question Type:   Photon energy
A monochromatic violet light of wavelength
390 nm is emitted by a light source with a
power of 200 Watt.

Q37: How many photons leave this
light source in a 10 second interval?

 P = W/t = E/t  E = Pt = (200)(10)
   = 2000 J
 Each photon has Energy = hc/λ =
   = (6.63 x 10-34)(3.0 x 108)/(390 x 10-9)
   = 5.1 x 10-19 J
 No of photons in 2000 J = 2000/(5.1 x 10-19)
  = 3.92 x 1021

 Q38: What is the momentum of
 each of these photons?

  p = h/λ = 1.7 x 10-27 kgms-1
Chapter 5
•   Topics covered:
•   Quantised Energy Levels for Atoms.
•   Emission Spectra
•   Absorption Spectra
•   Solar Spectrum.
•   Quantised Energy Levels
•   Wave Particle Duality.
5.0 Quantised Energy
                 Levels for Atoms
•   The Bohr model for the atom,
    restricting electrons to                         The 1st five
    certain regions or orbits with                   electron shells
    fixed energy levels, worked                      with their standing
    well in describing atomic
    structure and behaviour.                         waves
•   But it was not until DeBroglie
    suggested electrons had a
    wavelength, was there a
    satisfactory explanation of
    why fixed orbits and energies
    existed.
•   DeBroglie suggested that for         Disallowed orbit
    an electron to survive in an
    orbit, it must form a standing
    wave which represents a
    stable state for the electron.
•   Only certain radius orbits
    allow the standing wave to be
    set up, so electrons can only
    exist certain regions.

                                     Allowed Orbit
5.1 Emission Line Spectra
When atoms of substances are exposed to    The energy absorbed corresponds
certain kinds of energy sources, the       to jumps between the fixed energy
electrons surrounding the nucleus can      levels of the electron shells within
gain energy and move up to higher energy   the atom
levels, called “excited states”.
                                                  The excited electrons
                                                  return to their ground
                                                  state by emitting a
                                                  photon of a certain
                                                  frequency (colour)

The emitted photons from all electrons transitions
form “a picture” called an “Emission Spectrum”.
These Spectra are unique to each element and
are used to identify unknowns in a field called
spectroscopy.
    Helium Emission Spectra                    Sodium Emission Spectra
Light & Matter Revision
Question Type: Emission Spectra
A class looks at spectra from two
sources.                             Q40: State the electron mechanism,
i. an incandescent light globe       in each of the sources below, that
ii. a mercury vapour lamp            produces each spectrum.
They observe that the spectra are    i. Incandescent light globe
of different types.                  ii. Mercury vapour lamp

Q39: State the type of spectrum      i Thermal motion of free electrons
seen from each source.
i. Incandescent light globe          ii Energy transition of bound electrons
ii. Mercury vapour lamp

 i Continuous (or broad)
 ii Discrete (or individual lines)
5.2 Absorption Line Spectra
                                          If you look at the spectrum
                                          from a common incandescent
                                          light bulb, you'll see a
                                          relatively smooth spectrum
                                          with no part being brighter or
                                          darker than any other part.
                                          This type of a spectrum is
                                          called a continuous spectrum.
Electrons in an atom can jump between
                                          The pattern of absorption lines in
discreet energy levels by absorbing a
                                          a spectrum will be unique to a
photon.
                                          chemical element so we can use
So certain atoms under certain
                                          absorption lines to detect the
conditions will absorb very specific
                                          presence of specific elements in
wavelengths of light dependent on the
                                          astronomical objects.
configuration of their electrons.
This will remove energy and leave blank   An emission spectrum is simply the
spaces (dark lines) in the spectrum.      reverse of an absorption spectrum.
Light & Matter Revision
Question Type: Absorption Spectra
Figure 5a shows part of the emission
spectrum of hydrogen in more detail.

With a spectroscope, Val examines the
spectrum of light from the sun. The
spectrum is continuous, with colours
ranging from red to violet. However
there were black lines in the spectrum,
as shown in Figure 5b.

Q41: Explain why these dark lines are
present in the spectrum from the sun.

 The dark lines exist because photons are
 absorbed which correspond to the energy
 levels in hydrogen. This indicates the
 presence of hydrogen.
5.3 The Solar Spectrum
              Solar Spectrum                 When the “White Light” arriving at
                                             the Earth’s surface is passed
                                             through a prism, it is broken up into
                                             its constituent colours.
                                             This produces the “solar spectrum”




                                                It was later discovered that each
In 1802 English chemist William Wollaston was chemical element was
the first to note the appearance of a number of associated with a set of spectral
dark lines in the solar spectrum.               lines, and that the dark lines in
                                                the solar spectrum were caused
In 1814, Joseph Von Fraunhofer                  by absorption by those elements
independently rediscovered the                  in the upper layers of the sun.
lines and began a systematic                    Some of the observed features
study and careful measurement of                are also caused by absorption in
the wavelength of these features.               oxygen molecules in Earth’s
   In all, he mapped over 570 lines.            atmosphere.
5.4 Quantised Energy Levels
                                            The energy is associated with
Electrons surrounding the nucleus           these “jumps” is given by E = hf
can “jump” up and down between              where
allowed energy levels by absorbing          E = Energy of the emitted photon
or emitting a photon.                       for a downward jump OR the
The ABSORPTION and EMISSION                 energy absorbed from an incident
SPECTRA for atoms show the                  photon for an upward jump. (eV)
energy values associated with the                    h = Planck’s Constant
“jumps”.                                             f = Frequency of Photon
                                            (Hz)
              Mercury Energy Levels

                    Ionisation
    10.4 eV                           n=5
     8.8 eV                           n=4
     6.7 eV                           n=3
                                                 Electronsabsorbing State5(n = 3 can of
                                                     States in = 2 to n = represent
                                                      The n excited state n 10.4
                                                 Electrons Ground more than = 1) eV
     4.9 eV                           n=2        energy from a photon1.8 eV by: be
                                                  represents thea state electron
                                                  2. By emitting lowest photon
                                                 return to the groundcollision willfor
                                                        allowed energy levels
                                                 stripped completely4.9 eV photon
                                                    Emitting a single 6. 7the Mercury
                                                      electrons a from eV atoms.
                                                 1. followed byin Mercuryphoton
                                                             energy level
                                                 atom leaving it ionised
                                                                  OR
      0 eV                            n=1
                  Ground State
Light & Matter Revision
Question Type: Energy Levels
The spectrum of photons emitted by        Q42: What is the lowest energy
excited atoms is being investigated.      photon that could be emitted
Shown in Figure 6 is the atomic energy    from the excited atoms?
level diagram of the particular atom      The smallest energy gap was
being studied. Although most of the       that from n = 3 to n = 2, an
atoms are in the ground state, some       energy difference of 1.8 eV.
atoms are known to be in n = 2 and n =
                                         Q43: Calculate the wavelength of
3 excited states.
                                         the photon emitted when the atom
                                         changes from the n = 2 state to the
                                         ground state (n = 1).
                                         Data: h = 4.14 × 10-15 eV s ,
                                         c = 3.0 × 108 m s-1
                                         The energy of the photon was 3.4 eV.
                                         By substituting this into the equation
                                         E =hc/λ,
                                         the wavelength was 3.65 x 10-7 m.
Light & Matter Revision
Question Type: Energy Levels
Figure 2 shows the energy levels
of a sodium atom.                      A sodium atom is initially in an n = 4
                                       excited state.
                                       Q44: Calculate the highest frequency
                                       of light that this sodium atom could
                                       emit.
                                        ΔE = 3.61 = h f
                                        Hence f = 8.7 x 1014 Hz



Figure 2 shows that electrons in a      Electrons have an equivalent or de
sodium atom can only occupy specific    Broglie wavelength.
energy levels.
Q45: Describe how the wave nature of    Only orbits of an integral number of
electrons can explain this.             wavelengths are allowed as these
                                        will form a standing wave
Light & Matter Revision
Question Type: Energy Level Diagrams
Part of the visible region of the
spectrum of light emitted from excited
hydrogen gas has three lines as shown
in Figure 3.
The energy level diagram for the
hydrogen atom is shown in Figure 4.
The binding energy is 13.6 eV.
                                         Q46: What is the energy of the
                                         photons with a wavelength 434.1
                                         nm in Figure 3?
                                         E = hc/λ
                                           = 2.86 eV
                                         Q47: A different photon has an energy
                                         of 3.0 eV.
                                         On Figure 4 indicate with an arrow the
                                         electron transition that leads to
                                         emission of a photon of light with this
                                         energy.
5.5 The Wave - Particle Duality
•   As you can see from what has been
    presented, attempting to
    characterise light as only wave like
    or only particle like in nature has
    failed.
•   In fact, as of today, light (and matter)
    are regarded as some part particle
    like and some part wave like in
    nature.
•   This conflict is summarised in the
    use of the term “wave-particle
    duality” to describe light’s known         BUT THIS IS WHAT PHYSICS IS ALL
    behaviour.                                 ABOUT –
•   This is an unsatisfactory situation,       PUSHING THE ENVELOPE –
    as it defies our need for simple all       EXPLORING THE UNKNOWN –
    encompassing explanations for the          TRYING TO FIT RATIONAL AND LOGICAL
    behaviours we observe around us
    and in the universe.                       EXPLANATIONS TO THAT WHICH HAS
                                               BEEN OBSERVED.
•   It would appear, at this stage, we
    have not yet unravelled all the details
    of the behaviour of light and matter.
Ollie Leitl 2005

Mais conteúdo relacionado

Mais procurados

Interference Of Light
Interference Of LightInterference Of Light
Interference Of LightBPGCW
 
Class 12th Physics wave optics ppt part 2
Class 12th Physics wave optics ppt part 2 Class 12th Physics wave optics ppt part 2
Class 12th Physics wave optics ppt part 2 Arpit Meena
 
Ap2 unit6 open stax notes wave optics
Ap2 unit6 open stax notes wave opticsAp2 unit6 open stax notes wave optics
Ap2 unit6 open stax notes wave opticsSarah Sue Calbio
 
9.3 interference
9.3 interference9.3 interference
9.3 interferencePaula Mills
 
Class 12th Physics wave optics ppt
Class 12th Physics wave optics pptClass 12th Physics wave optics ppt
Class 12th Physics wave optics pptArpit Meena
 
A project on wave nature of light
A project on wave nature of lightA project on wave nature of light
A project on wave nature of lightKANNAN
 
Tunay na presentation sa physics
Tunay na presentation sa physicsTunay na presentation sa physics
Tunay na presentation sa physicsDith Jose
 
7m dual nature_of_matter__radiation
7m dual nature_of_matter__radiation7m dual nature_of_matter__radiation
7m dual nature_of_matter__radiationPrayash Mohapatra
 

Mais procurados (16)

Interference of light
Interference of lightInterference of light
Interference of light
 
Interference Of Light
Interference Of LightInterference Of Light
Interference Of Light
 
Class 12th Physics wave optics ppt part 2
Class 12th Physics wave optics ppt part 2 Class 12th Physics wave optics ppt part 2
Class 12th Physics wave optics ppt part 2
 
Gensci reviewer 3 rd qrtr k 12
Gensci reviewer 3 rd qrtr k 12Gensci reviewer 3 rd qrtr k 12
Gensci reviewer 3 rd qrtr k 12
 
Ap2 unit6 open stax notes wave optics
Ap2 unit6 open stax notes wave opticsAp2 unit6 open stax notes wave optics
Ap2 unit6 open stax notes wave optics
 
9.3 interference
9.3 interference9.3 interference
9.3 interference
 
Notes1
Notes1Notes1
Notes1
 
Wave optics
Wave opticsWave optics
Wave optics
 
Class 12th Physics wave optics ppt
Class 12th Physics wave optics pptClass 12th Physics wave optics ppt
Class 12th Physics wave optics ppt
 
A project on wave nature of light
A project on wave nature of lightA project on wave nature of light
A project on wave nature of light
 
Chapter 5 diffraction
Chapter 5 diffractionChapter 5 diffraction
Chapter 5 diffraction
 
Tunay na presentation sa physics
Tunay na presentation sa physicsTunay na presentation sa physics
Tunay na presentation sa physics
 
Physics 1a Summary
Physics 1a SummaryPhysics 1a Summary
Physics 1a Summary
 
Topic 6 EM waves
Topic 6 EM wavesTopic 6 EM waves
Topic 6 EM waves
 
Chapter 7
Chapter 7Chapter 7
Chapter 7
 
7m dual nature_of_matter__radiation
7m dual nature_of_matter__radiation7m dual nature_of_matter__radiation
7m dual nature_of_matter__radiation
 

Destaque

Ch15 - light history
Ch15  - light historyCh15  - light history
Ch15 - light historycpphysics
 
Light its nature and behaviour By Malik Shahroz
Light its nature and behaviour By Malik ShahrozLight its nature and behaviour By Malik Shahroz
Light its nature and behaviour By Malik ShahrozMalik Akber
 
The nature and propagation of light
The nature and propagation of lightThe nature and propagation of light
The nature and propagation of lightJessabeth Aluba
 
Interference and the Wave Nature of Light
Interference and the Wave Nature of LightInterference and the Wave Nature of Light
Interference and the Wave Nature of LightTaimoor Muzaffar Gondal
 
Theories of light
Theories of lightTheories of light
Theories of lightJumaed
 
Particle theory of light
Particle theory of lightParticle theory of light
Particle theory of lightAnna Martinez
 
Photoelectric Effect And Dual Nature Of Matter And Radiation Class 12
Photoelectric Effect And Dual Nature Of Matter And Radiation Class 12Photoelectric Effect And Dual Nature Of Matter And Radiation Class 12
Photoelectric Effect And Dual Nature Of Matter And Radiation Class 12Self-employed
 
Photoelectric effect ppt
Photoelectric effect pptPhotoelectric effect ppt
Photoelectric effect pptJason Baughman
 

Destaque (14)

Ch15 - light history
Ch15  - light historyCh15  - light history
Ch15 - light history
 
5 Nature of light
5 Nature of light5 Nature of light
5 Nature of light
 
Light its nature and behaviour By Malik Shahroz
Light its nature and behaviour By Malik ShahrozLight its nature and behaviour By Malik Shahroz
Light its nature and behaviour By Malik Shahroz
 
Light dualism
Light dualismLight dualism
Light dualism
 
Newton
NewtonNewton
Newton
 
Hp 15 win
Hp 15 winHp 15 win
Hp 15 win
 
The nature and propagation of light
The nature and propagation of lightThe nature and propagation of light
The nature and propagation of light
 
The nature of light
The nature of lightThe nature of light
The nature of light
 
Interference and the Wave Nature of Light
Interference and the Wave Nature of LightInterference and the Wave Nature of Light
Interference and the Wave Nature of Light
 
Theories of light
Theories of lightTheories of light
Theories of light
 
Particle theory of light
Particle theory of lightParticle theory of light
Particle theory of light
 
Nature of light (2)
Nature of light (2)Nature of light (2)
Nature of light (2)
 
Photoelectric Effect And Dual Nature Of Matter And Radiation Class 12
Photoelectric Effect And Dual Nature Of Matter And Radiation Class 12Photoelectric Effect And Dual Nature Of Matter And Radiation Class 12
Photoelectric Effect And Dual Nature Of Matter And Radiation Class 12
 
Photoelectric effect ppt
Photoelectric effect pptPhotoelectric effect ppt
Photoelectric effect ppt
 

Semelhante a Vu4 light&matter2009

Semelhante a Vu4 light&matter2009 (20)

Vu4 light&matter2009
Vu4 light&matter2009Vu4 light&matter2009
Vu4 light&matter2009
 
chapter2.pdf
chapter2.pdfchapter2.pdf
chapter2.pdf
 
Optics 09 april 2021
Optics 09 april 2021Optics 09 april 2021
Optics 09 april 2021
 
Light waves
Light wavesLight waves
Light waves
 
Unit iii (advances in metrology)
Unit iii (advances in metrology)Unit iii (advances in metrology)
Unit iii (advances in metrology)
 
unit iii.pptx
unit iii.pptxunit iii.pptx
unit iii.pptx
 
Double Slit Experiment
Double Slit ExperimentDouble Slit Experiment
Double Slit Experiment
 
Unit III
Unit IIIUnit III
Unit III
 
Optics
Optics Optics
Optics
 
Wave particle duality of light- A changing Notion in Science
Wave particle duality of light- A changing Notion in ScienceWave particle duality of light- A changing Notion in Science
Wave particle duality of light- A changing Notion in Science
 
Wave particle duality
Wave particle dualityWave particle duality
Wave particle duality
 
Interference original
Interference originalInterference original
Interference original
 
Production of xray
Production of xrayProduction of xray
Production of xray
 
Module No. 45
Module No. 45Module No. 45
Module No. 45
 
4
44
4
 
Basic Idea of Laser by deepika gupta
Basic Idea of Laser by deepika guptaBasic Idea of Laser by deepika gupta
Basic Idea of Laser by deepika gupta
 
class22A.ppt
class22A.pptclass22A.ppt
class22A.ppt
 
ELECTRO MAGNETIC RAIATIONS.pptx
ELECTRO MAGNETIC RAIATIONS.pptxELECTRO MAGNETIC RAIATIONS.pptx
ELECTRO MAGNETIC RAIATIONS.pptx
 
waves
waveswaves
waves
 
1_ Wave Optics_SKV.pptx
1_ Wave Optics_SKV.pptx1_ Wave Optics_SKV.pptx
1_ Wave Optics_SKV.pptx
 

Mais de Andrew Grichting

VCE Physics Unit 3: Electronics & Photonics Base notes
VCE Physics Unit 3: Electronics & Photonics Base notesVCE Physics Unit 3: Electronics & Photonics Base notes
VCE Physics Unit 3: Electronics & Photonics Base notesAndrew Grichting
 
Empirical Research Activity (ERA) Guide
Empirical Research Activity (ERA) GuideEmpirical Research Activity (ERA) Guide
Empirical Research Activity (ERA) GuideAndrew Grichting
 
VCE Physics: Dealing with numerical measurments
VCE Physics: Dealing with numerical measurmentsVCE Physics: Dealing with numerical measurments
VCE Physics: Dealing with numerical measurmentsAndrew Grichting
 
VCE Physics: Analysis of experiments
VCE Physics: Analysis of experimentsVCE Physics: Analysis of experiments
VCE Physics: Analysis of experimentsAndrew Grichting
 
Pedigree Charts Powerpoint
Pedigree Charts PowerpointPedigree Charts Powerpoint
Pedigree Charts PowerpointAndrew Grichting
 

Mais de Andrew Grichting (12)

VCE Physics Unit 3: Electronics & Photonics Base notes
VCE Physics Unit 3: Electronics & Photonics Base notesVCE Physics Unit 3: Electronics & Photonics Base notes
VCE Physics Unit 3: Electronics & Photonics Base notes
 
Empirical Research Activity (ERA) Guide
Empirical Research Activity (ERA) GuideEmpirical Research Activity (ERA) Guide
Empirical Research Activity (ERA) Guide
 
Introduction to evolution
Introduction to evolutionIntroduction to evolution
Introduction to evolution
 
Hypothesis formation
Hypothesis formationHypothesis formation
Hypothesis formation
 
Survival of the sneakiest
Survival of the sneakiestSurvival of the sneakiest
Survival of the sneakiest
 
VCE Physics: Dealing with numerical measurments
VCE Physics: Dealing with numerical measurmentsVCE Physics: Dealing with numerical measurments
VCE Physics: Dealing with numerical measurments
 
VCE Physics: Analysis of experiments
VCE Physics: Analysis of experimentsVCE Physics: Analysis of experiments
VCE Physics: Analysis of experiments
 
Fukushima daiichiareva
Fukushima daiichiarevaFukushima daiichiareva
Fukushima daiichiareva
 
Matrices 2
Matrices 2Matrices 2
Matrices 2
 
Matrices 1
Matrices 1Matrices 1
Matrices 1
 
Power Laws
Power LawsPower Laws
Power Laws
 
Pedigree Charts Powerpoint
Pedigree Charts PowerpointPedigree Charts Powerpoint
Pedigree Charts Powerpoint
 

Último

What's New in Teams Calling, Meetings and Devices March 2024
What's New in Teams Calling, Meetings and Devices March 2024What's New in Teams Calling, Meetings and Devices March 2024
What's New in Teams Calling, Meetings and Devices March 2024Stephanie Beckett
 
Powerpoint exploring the locations used in television show Time Clash
Powerpoint exploring the locations used in television show Time ClashPowerpoint exploring the locations used in television show Time Clash
Powerpoint exploring the locations used in television show Time Clashcharlottematthew16
 
My INSURER PTE LTD - Insurtech Innovation Award 2024
My INSURER PTE LTD - Insurtech Innovation Award 2024My INSURER PTE LTD - Insurtech Innovation Award 2024
My INSURER PTE LTD - Insurtech Innovation Award 2024The Digital Insurer
 
Commit 2024 - Secret Management made easy
Commit 2024 - Secret Management made easyCommit 2024 - Secret Management made easy
Commit 2024 - Secret Management made easyAlfredo García Lavilla
 
Dev Dives: Streamline document processing with UiPath Studio Web
Dev Dives: Streamline document processing with UiPath Studio WebDev Dives: Streamline document processing with UiPath Studio Web
Dev Dives: Streamline document processing with UiPath Studio WebUiPathCommunity
 
WordPress Websites for Engineers: Elevate Your Brand
WordPress Websites for Engineers: Elevate Your BrandWordPress Websites for Engineers: Elevate Your Brand
WordPress Websites for Engineers: Elevate Your Brandgvaughan
 
Designing IA for AI - Information Architecture Conference 2024
Designing IA for AI - Information Architecture Conference 2024Designing IA for AI - Information Architecture Conference 2024
Designing IA for AI - Information Architecture Conference 2024Enterprise Knowledge
 
Bun (KitWorks Team Study 노별마루 발표 2024.4.22)
Bun (KitWorks Team Study 노별마루 발표 2024.4.22)Bun (KitWorks Team Study 노별마루 발표 2024.4.22)
Bun (KitWorks Team Study 노별마루 발표 2024.4.22)Wonjun Hwang
 
Vector Databases 101 - An introduction to the world of Vector Databases
Vector Databases 101 - An introduction to the world of Vector DatabasesVector Databases 101 - An introduction to the world of Vector Databases
Vector Databases 101 - An introduction to the world of Vector DatabasesZilliz
 
Kotlin Multiplatform & Compose Multiplatform - Starter kit for pragmatics
Kotlin Multiplatform & Compose Multiplatform - Starter kit for pragmaticsKotlin Multiplatform & Compose Multiplatform - Starter kit for pragmatics
Kotlin Multiplatform & Compose Multiplatform - Starter kit for pragmaticscarlostorres15106
 
Search Engine Optimization SEO PDF for 2024.pdf
Search Engine Optimization SEO PDF for 2024.pdfSearch Engine Optimization SEO PDF for 2024.pdf
Search Engine Optimization SEO PDF for 2024.pdfRankYa
 
AI as an Interface for Commercial Buildings
AI as an Interface for Commercial BuildingsAI as an Interface for Commercial Buildings
AI as an Interface for Commercial BuildingsMemoori
 
Are Multi-Cloud and Serverless Good or Bad?
Are Multi-Cloud and Serverless Good or Bad?Are Multi-Cloud and Serverless Good or Bad?
Are Multi-Cloud and Serverless Good or Bad?Mattias Andersson
 
DevEX - reference for building teams, processes, and platforms
DevEX - reference for building teams, processes, and platformsDevEX - reference for building teams, processes, and platforms
DevEX - reference for building teams, processes, and platformsSergiu Bodiu
 
Gen AI in Business - Global Trends Report 2024.pdf
Gen AI in Business - Global Trends Report 2024.pdfGen AI in Business - Global Trends Report 2024.pdf
Gen AI in Business - Global Trends Report 2024.pdfAddepto
 
Transcript: New from BookNet Canada for 2024: BNC CataList - Tech Forum 2024
Transcript: New from BookNet Canada for 2024: BNC CataList - Tech Forum 2024Transcript: New from BookNet Canada for 2024: BNC CataList - Tech Forum 2024
Transcript: New from BookNet Canada for 2024: BNC CataList - Tech Forum 2024BookNet Canada
 
Integration and Automation in Practice: CI/CD in Mule Integration and Automat...
Integration and Automation in Practice: CI/CD in Mule Integration and Automat...Integration and Automation in Practice: CI/CD in Mule Integration and Automat...
Integration and Automation in Practice: CI/CD in Mule Integration and Automat...Patryk Bandurski
 
"Federated learning: out of reach no matter how close",Oleksandr Lapshyn
"Federated learning: out of reach no matter how close",Oleksandr Lapshyn"Federated learning: out of reach no matter how close",Oleksandr Lapshyn
"Federated learning: out of reach no matter how close",Oleksandr LapshynFwdays
 
Nell’iperspazio con Rocket: il Framework Web di Rust!
Nell’iperspazio con Rocket: il Framework Web di Rust!Nell’iperspazio con Rocket: il Framework Web di Rust!
Nell’iperspazio con Rocket: il Framework Web di Rust!Commit University
 

Último (20)

What's New in Teams Calling, Meetings and Devices March 2024
What's New in Teams Calling, Meetings and Devices March 2024What's New in Teams Calling, Meetings and Devices March 2024
What's New in Teams Calling, Meetings and Devices March 2024
 
Powerpoint exploring the locations used in television show Time Clash
Powerpoint exploring the locations used in television show Time ClashPowerpoint exploring the locations used in television show Time Clash
Powerpoint exploring the locations used in television show Time Clash
 
My INSURER PTE LTD - Insurtech Innovation Award 2024
My INSURER PTE LTD - Insurtech Innovation Award 2024My INSURER PTE LTD - Insurtech Innovation Award 2024
My INSURER PTE LTD - Insurtech Innovation Award 2024
 
Commit 2024 - Secret Management made easy
Commit 2024 - Secret Management made easyCommit 2024 - Secret Management made easy
Commit 2024 - Secret Management made easy
 
Dev Dives: Streamline document processing with UiPath Studio Web
Dev Dives: Streamline document processing with UiPath Studio WebDev Dives: Streamline document processing with UiPath Studio Web
Dev Dives: Streamline document processing with UiPath Studio Web
 
WordPress Websites for Engineers: Elevate Your Brand
WordPress Websites for Engineers: Elevate Your BrandWordPress Websites for Engineers: Elevate Your Brand
WordPress Websites for Engineers: Elevate Your Brand
 
Designing IA for AI - Information Architecture Conference 2024
Designing IA for AI - Information Architecture Conference 2024Designing IA for AI - Information Architecture Conference 2024
Designing IA for AI - Information Architecture Conference 2024
 
Bun (KitWorks Team Study 노별마루 발표 2024.4.22)
Bun (KitWorks Team Study 노별마루 발표 2024.4.22)Bun (KitWorks Team Study 노별마루 발표 2024.4.22)
Bun (KitWorks Team Study 노별마루 발표 2024.4.22)
 
Vector Databases 101 - An introduction to the world of Vector Databases
Vector Databases 101 - An introduction to the world of Vector DatabasesVector Databases 101 - An introduction to the world of Vector Databases
Vector Databases 101 - An introduction to the world of Vector Databases
 
Kotlin Multiplatform & Compose Multiplatform - Starter kit for pragmatics
Kotlin Multiplatform & Compose Multiplatform - Starter kit for pragmaticsKotlin Multiplatform & Compose Multiplatform - Starter kit for pragmatics
Kotlin Multiplatform & Compose Multiplatform - Starter kit for pragmatics
 
Search Engine Optimization SEO PDF for 2024.pdf
Search Engine Optimization SEO PDF for 2024.pdfSearch Engine Optimization SEO PDF for 2024.pdf
Search Engine Optimization SEO PDF for 2024.pdf
 
AI as an Interface for Commercial Buildings
AI as an Interface for Commercial BuildingsAI as an Interface for Commercial Buildings
AI as an Interface for Commercial Buildings
 
Are Multi-Cloud and Serverless Good or Bad?
Are Multi-Cloud and Serverless Good or Bad?Are Multi-Cloud and Serverless Good or Bad?
Are Multi-Cloud and Serverless Good or Bad?
 
DevEX - reference for building teams, processes, and platforms
DevEX - reference for building teams, processes, and platformsDevEX - reference for building teams, processes, and platforms
DevEX - reference for building teams, processes, and platforms
 
E-Vehicle_Hacking_by_Parul Sharma_null_owasp.pptx
E-Vehicle_Hacking_by_Parul Sharma_null_owasp.pptxE-Vehicle_Hacking_by_Parul Sharma_null_owasp.pptx
E-Vehicle_Hacking_by_Parul Sharma_null_owasp.pptx
 
Gen AI in Business - Global Trends Report 2024.pdf
Gen AI in Business - Global Trends Report 2024.pdfGen AI in Business - Global Trends Report 2024.pdf
Gen AI in Business - Global Trends Report 2024.pdf
 
Transcript: New from BookNet Canada for 2024: BNC CataList - Tech Forum 2024
Transcript: New from BookNet Canada for 2024: BNC CataList - Tech Forum 2024Transcript: New from BookNet Canada for 2024: BNC CataList - Tech Forum 2024
Transcript: New from BookNet Canada for 2024: BNC CataList - Tech Forum 2024
 
Integration and Automation in Practice: CI/CD in Mule Integration and Automat...
Integration and Automation in Practice: CI/CD in Mule Integration and Automat...Integration and Automation in Practice: CI/CD in Mule Integration and Automat...
Integration and Automation in Practice: CI/CD in Mule Integration and Automat...
 
"Federated learning: out of reach no matter how close",Oleksandr Lapshyn
"Federated learning: out of reach no matter how close",Oleksandr Lapshyn"Federated learning: out of reach no matter how close",Oleksandr Lapshyn
"Federated learning: out of reach no matter how close",Oleksandr Lapshyn
 
Nell’iperspazio con Rocket: il Framework Web di Rust!
Nell’iperspazio con Rocket: il Framework Web di Rust!Nell’iperspazio con Rocket: il Framework Web di Rust!
Nell’iperspazio con Rocket: il Framework Web di Rust!
 

Vu4 light&matter2009

  • 1. VCE Physics Unit 4 Topic 2 Interactions of Light & Matter
  • 2. Unit Outline To achieve the outcome the student should demonstrate the knowledge and skills to : • Explain the production of incoherent light from wide spectrum light sources including the Sun, light bulbs and candles (descriptive) in terms of thermal motion of electrons. • Explain the results of Young’s double slit experiment as evidence for the wave like nature of light including: – constructive and destructive interference in terms of path difference – qualitative effect of wavelength on interference patterns • Interpret the pattern produced by light when it passes through a gap or past an obstacle in terms of the diffraction of waves and the significance of the magnitude of the λ/w ratio • interpret the photoelectric effect as evidence of the particle like nature of light including the KE of emitted photoelectrons in terms of the energy of incident photons Ek max = hf – W, using energy units of both joules and electron-volts, effects of intensity of incident irradiation on the emission of photoelectrons. • interpret electron diffraction patterns as evidence of the wavelike nature of matter expressed as the De Broglie wavelength λ = h/p • compare momentum of photons and of particles of the same wavelength including calculations using p = h/λ • interpret atomic absorption and emission spectra, including those from metal vapour lamps in terms of the quantised energy level model of the atom, including calculations of the energy of photons emitted or absorbed. ∆E = hf • explain a model of quantised energy levels of the atom in which electrons are found in standing wave states • use safe and responsible practises when working with light sources, lasers and related equipment.
  • 3. Chapter 1 Topics covered: • The Nature of Light. • Interference. • Incoherent Light. • Coherent Light • Young’s Experiment. • Path Difference • Single Slit Diffraction • Diffraction around Objects
  • 4. 1.0 The Nature of Light EMR is a self propagating wave Light is a form of ENERGY. consisting of mutually It is described as ELECTRO - perpendicular, varying MAGNETIC RADIATION (EMR). ELECTRIC and MAGNETIC FIELDS. EMR travels through a vacuum at 300,000 kms-1, (3.0 x 108 ms-1) Changing Magnetic Field Direction of Electromagnetic Wave Movement Changing Electric Field
  • 5. 1.1 Superposition Single waves, called Once the pulses. have the ability superposition is to pass through one complete the pulses another and, while continue their occupying the same journey unaffected. space, add together in a process called A series of pulses SUPERPOSITION. together form a Trough Crest wave train with alternating crests When two wave trains interact and troughs with one another they also Constructive interference occurs when undergo SUPERPOSITION and the two wave trains are in phase will either: 1. Add together to produce a larger wave - a process called CONSTRUCTIVE INTERFERENCE or 2. Subtract from one another to Destructive interference occurs when produce no wave – a process called the two wave trains are 1800 out of DESTRUCTIVE INTERFERENCE phase
  • 6. 1.2 Interference Light behaves in a similar manner. When light (with certain properties) Destructive is passed through two narrow slits, Interference an “interference pattern” is Constructive produced, showing constructive Interference (light bands) and destructive (dark bands) interference. Bright bands occur where a crest and a Waves, in this case water crest (or a trough and a waves, when passed trough) arrive at the through two narrow slits, screen at the same time “interfere” or interact with one another to produce areas of large disturbances Dark bands Crest Bright (Constructive Interference) Band occur when a Trough or areas of no disturbance crest and a (Destructive Interference) trough arrive at Dark the screen at Band the same time
  • 7. 1.3 Incoherent Light Light is generated by luminous bodies, eg, The Sun, light globes, burning candles. Light is produced when atoms of the filaments or source become electrically excited and produce an electromagnetic or light wave. Since the excitations occur in an unpredictably random fashion, the Typical light sources such as those light waves are NOT produced in mentioned above have an inbuilt regular repeating manner and so do irregularity in the way they produce light. not maintain a constant “phase relationship” with one other. About once every 10-8 sec, a source will randomly alter its phase. Incoherent light, when combined, This leads to the sources giving off a produces rapidly moving areas of broad spectrum of white light composed constructive and destructive of all colours in the rainbow. interference and therefore do not Each of the millions of colours have produce a stable, visible interference waves that are random to each other. pattern. This is called INCOHERENT LIGHT.
  • 8. Light & Matter Revision Question Type: Incandescent Light Q1: The light from a candle can best be described as A. coherent, arising from the vibrations of electrons. B. incoherent, arising only from the transition of electrons in excited energy levels falling to lower energy levels. C. coherent, arising only from the transition of electrons in excited energy levels falling to lower energy levels. D. incoherent, arising from the vibrations of electrons.
  • 9. Light & Matter Revision Question Type: Incandescent Light The spectrum of wavelengths produced by a particular incandescent light globe is shown in Figure 1 below. Q2: The light produced by an incandescent light globe can best be described as A. coherent. B. incoherent. C. monochromatic. D. in phase. Q3: Describe the mechanism by which light is produced in an incandescent light globe. The thermal/random excitation of electrons in the filament leads to the emission of a broad/continuous spectrum.
  • 10. 1.4 Coherent Light Coherence is a property of waves that measures their ability to interfere with Coherent waves (zero phase each other. difference) Shown are monochromatic (single colour) light waves of the same frequency. Coherent waves (constant They are coherent and in phase, and will phase difference) combine constructively to produce bright Two waves that are coherent can white light be combined to produce an unmoving distribution of constructive and destructive interference (a visible interference pattern) depending on the relative phase of the waves at their meeting point. Lasers generate light at a single wave length and frequency and all of the waves (and PHOTONS) are in phase. This is called COHERENT LIGHT.
  • 11. Light & Matter Revision Question Type: Coherent Light Q4: Which of the following light sources will produce coherent, monochromatic light A. Sunlight B. LED C. Light globe with filter D. LASER
  • 12. 1.5 Young’s Experiment First performed by This pattern has a series of equally Thomas Young Thomas Young in the spaced coloured and black bands 1773 – 1829 early 1800’s this spread across the screen onto experiment proved which it is projected. light was a wave. The width of the coloured bands It has been voted the and their spacing depends on most elegant Incident Light the wavelength of the light used. experiment ever Short wavelength, BLUE light devised. produces a pattern with narrow blue When light of a single frequency bands which are closely spaced. (colour) is passed through a pair Long wavelength, RED light of closely spaced, narrow slits produces a pattern with wider an “interference pattern” is red bands which are spread produced. farther apart. BLUE Light using the same slits Screen Double Slits RED Light
  • 13. Light & Matter Revision Question Type: Young’s Experiment Thomas Young’s double slit experiment The slits are now moved further has been replicated in the experimental from the screen. arrangement shown in Figure 4. Q6: What effect would this have on the pattern observed on the screen? The pattern will spread further across the screen. Q5: Explain using the wave theory of light why a series of bright and dark bars are observed on the screen. Slits provide 2 coherent sources leading to superposition which produces constructive and destructive interference leading to bright and dark bars on screen
  • 14. Light & Matter Revision Question Type: Young’s Experiment A physics teacher has apparatus to show Young’s double slit experiment. The apparatus is shown in Figure 4. The pattern of bright and dark bands is observed on the screen. Q7: Which one of the following actions will increase the distance, Δx, between dark bands in this double slit interference pattern? A. decrease the slit width B. decrease the slit separation C. decrease the slit screen distance D. decrease the wavelength of the light
  • 15. 1.6 Path Difference Light travels the same distance from S1 and S2 to reach the central bright band, BC The path difference The dark band (D1) on S1 S2 S1BC – S2BC = 0. either side of AC occurs So a crest and a crest because a crest and a (or trough and trough) trough are arriving at the arrive at AC at the same same time leading to time, leading to destructive interference. constructive Path difference: interference or a bright S1D1 – S2D1 = ½ λ band The next dark band (D2) Slits For the next bright has path difference = 1½λ S1 S2 band path difference S1B1 – S2B1 = 1 λ For the next bright So dark bands occur band path difference when would be: path difference = (n + ½)λ S1B2 – S2B2 = 2λ where n = So bright bands occur 0,1,2,3, etc when path difference = nλ where n = 0,1,2,3,etc D2 B1 D1 B C D1 B1 D2
  • 16. Light & Matter Revision Question Type: Interference – Path Difference In the following diagram, laser light of wavelength 600 nm is shone onto a pair of Q9: Estimate the difference in parallel slits and a pattern of alternating length between P1 and P2. light/dark bands is projected onto a wall. The indicated position is on the sixth antinode from the centre. The path difference is 6λ = 6 × 600nm = 3.6 ×10−6 m Q8: Explain how the observed pattern on the wall supports a wave model for light. The alternating pattern represents an interference pattern which is a wave phenomenon. The bright bands represent regions of constructive interference where the difference in path length from each source is a multiple of wavelengths. Waves therefore arrive from both sources in phase. The dark regions represent bands of destructive interference with the waves being half a wavelength out of phase due to path differences.
  • 17. Light & Matter Revision Question Type: Interference – Path Difference Q10: What is the difference in length (S2P2 - S1P2) where P2 is the second maximum away from the central axis. Path difference to 2nd max = 2λ = 5.6 cm Students have set up an experiment similar to that of English physicist Thomas Young. The students’ experiment uses microwaves of wavelength λ = 2.8 cm instead of light. The beam of microwaves passes through two narrow slits shown as S1 and S2 in Figure 3. The students measure the intensity of the resulting beam at points along the line shown and determine the positions of maximum intensity. These are shown as filled circles and marked P0, P1 . . .
  • 18. 1.7 Single Slit Diffraction Appreciable diffraction will occur if When light of a single THE EXTENT OF the ratio λ/w is between about 0.1 wavelength shines DIFFRACTION and 50. Outside this range, through a narrow gap or DEPENDS UPON diffraction is not observed. single slit, a “diffraction THE RATIO λ /w, pattern” is produced. WHERE λ = Diffraction patterns for blue and WAVELENGTH red light show that the shorter The pattern consists of AND w = SLIT wavelength blue light produces the a rather wide, coloured WIDTH. more “compact” pattern, while Screen central maximum with the longer wavelength red a series of thinner produces a more spread out coloured and dark pattern bands spreading out In other words, the spacing from the centre. between the lines is Width = w wavelength dependent and Single Slit patterns with the same line Incident Light spacing were made by light Wavelength = λ of the same wavelength BLUE light using same slit Note: The wider the slit the narrower central maximum RED light
  • 19. Light & Matter Revision Question Type: Single Slit Diffraction In an experiment, monochromatic laser light of wavelength 600 nm shines through a narrow slit, and the intensity of the transmitted light is recorded on the screen some distance away as shown in Figure 2a. The intensity pattern seen on the screen is shown in Figure 2b. Q11: Which one of the intensity patterns (A-D) below best indicates the pattern that would be seen if a wider slit was used?
  • 20. 1.8 Diffraction around Objects In addition to diffraction Sunlight diffracting around occurring when light passes a car tail light lens through gaps, it can also occur when light passes around objects The shadow of a Red Light hand holding a coin Diffracting around a illuminated by a He – You can see a pinhead Ne Laser. diffraction Point your hand pattern toward a light through your source and view fingers through this gap
  • 21. Chapter 2 Topics covered: • Models for Light Behaviour. • Particle like Properties of Light. • Wave like Properties of Light.
  • 22. 2.0 Models for Light Behaviour • Light is/has energy and thus Two separate models have can do work. We see this in been proposed: things like plants growing or 1. The Particle Model: the chemical reactions on This says that all properties of photographic films. light can be explained by • It is quite easy to list or show assuming light is made up of what light DOES but more a stream of individual difficult to say what light IS ! particles. • Much time and effort has been put into trying to find a 2. The Wave Model: THEORY or MODEL which This says that all properties of will explain ALL the light can be explained by properties and behaviours assuming light is made up of exhibited by light. a stream of waves.
  • 23. 2.1 Particle like Properties of Light • Most properties of light can be explained using the “Wave Model”. • However, some properties can only be explained by assuming light is made up of a “stream of particles”. 4. Certain colours of light can cause • These particle like properties electrons to be emitted from some are: metal surfaces. This is called the 1. Light exerts pressure. Photoelectric Effect. 2. Light can travel through a 5. Light reflects from shiny vacuum as well as through surfaces (both a wave and particle like transparent media. property). 6. Different colours correspond to 3. Beams of light are bent by different energies of light (both a wave gravity. and particle property).
  • 24. 2.2 Wave Like Properties of Light • Light exhibits many properties which can be demonstrated by other forms of waves eg. Water waves or waves on springs or strings. • These wavelike properties of light are: 4. Light undergoes Diffraction when passing through narrow gaps. 1. Light travels very fast, but its speed does depend upon the medium 5. Light beams can pass through one through which it moves. another unaffected. 2. Light undergoes Reflection according 6. Different colours of light correspond to the Laws of Reflection. to different energies. 3. Light undergoes Refraction according 7. Light forms interference patterns to the Laws of Refraction. when passed through a pair of closely spaced, narrow slits.
  • 25. Light & Matter Revision Question Type: Light Properties Q12. Light sometimes behaves as a particle and sometimes as a wave. Which one or more of the following properties does light sometimes show? A. mass B. momentum C. charge D. energy
  • 26. Light & Matter Revision Question Type: Models for Light Q 13: The table below contains some predictions for the behaviour of light incident on a shiny metal sheet. Complete the table by placing a .Y. (Yes) or .N. (No) in the appropriate boxes if the prediction is supported by the wave and/or particle model of light. Some answers have already been provided. It is possible for predictions to be supported by both models Y Y N N
  • 27. Chapter 3 Topics covered: • The Photoelectric Effect. • Electron Energies • Energy Units • Practical Applications
  • 28. 3.0 The Photoelectric Effect Albert Einstein won his Nobel The most important of the Prize for his explanation of this particle like properties of phenomena not for his work on light is the Photoelectric Relativity Effect. Simply put, the photoelectric effect occurs when light shines on a metal surface causing electrons to be ejected from the metal. There are, however, certain restrictions: (a) Only certain metals respond. (b) Light must be above a certain frequency - the Cut Off Frequency. Einstein called these individual light particles PHOTONS Louis De Broglie working in the 1920’s suggested a photon looked like this.
  • 29. 3.1 The Photoelectric Effect (2) The photoelectric effect is best studied Incident Photon Evacuated using the demonstration circuit shown. Glass Tube e An incident photon of the right frequency + (that is above the Cut Off Frequency) Ejected Electron striking a susceptible metal (such as aluminium, copper, lead, zinc and many Galvanometer G others), will free an electron. Rheostat The electron will cross the gap in the evacuated tube, due to electrostatic attraction. + So a stream of incident photons striking the metal will release a stream of electrons, setting up a current in the circuit which will be detected by the galvanometer (a sensitive ammeter able to detect currents in the μA range).
  • 30. 3.2 The Photoelectric Effect (3) The current (of ejected photoelectrons) does NOT depend upon the VOLTAGE between the plates. Incident Photon The size of the current DOES depend Evacuated upon the NUMBER of photons striking Glass Tube e the plate. + The NUMBER of photons depends Ejected Electron upon the INTENSITY of the incident light. Low Intensity or Dim Light of a Current (I) particular colour or frequency will produce a small current. Bright Light High Intensity or Bright Light of the same frequency will produce a large Dim Light current. Voltage (V)
  • 31. 3.3 The Photoelectric Effect (4) To study the energy possessed by the electrons ejected from the metal the Evacuated Evacuated Evacuated Glass Tube + Glass Tube Glass Tube + circuit is changed. + + + + The power supply has been reversed, + reversing the polarity of the plates. Some e-e-electrons crossenough No morenow don’t have the gap All still have enough thus energy to crossgap gap Current = 0 energy to cross the the Galvanometer G Galvanometer Moving the slider on the rheostat increases the voltage between the plates. This increasing voltage stops the least Rheostat Rheostat Rheostat energetic electrons crossing the gap, reducing the current in the circuit. + + The slider is again moved until no Current (I) electrons have enough energy to cross the gap and the current drops to zero. At this point the “Cut Off Voltage” (VC) has been reached. Voltage (V) VC
  • 32. 3.4 The Photoelectric Effect (5) The fact that the current falls to zero only slowly as the reverse voltage increases indicates the photoelectrons are ejected from the metal surface with VARYING AMOUNTS OF ENERGY. Different Intensities of the same Current (I) colour (frequency) have the same cut off voltages Different frequencies and/or -Vc Voltage (V) different metals will have differing cut off voltages
  • 33. Light & Matter Revision Question Type: Photoelectric effect A student carries out an experiment to Q14: What is the maximum kinetic investigate the photoelectric effect. She energy (in eV) of the electrons shines a monochromatic light onto a ejected from the plate P? metal plate (P) inside a sealed glass 1.7 eV chamber as shown in Figure 2. Q15: What is the subsequent maximum speed of these electrons ejected from plate P? KEMAX = 1.7 eV = (1.7)(1.6 x 10-19) J = 2.72 x 10-19 J KE = ½ mv2 v = 7.73 x 105 ms-1 The current in the circuit changes as the voltage is varied as shown in Figure 3.
  • 34. Light & Matter Revision Question Type: Photoelectric effect Blue light of wavelength 360 nm was Q16: Determine the threshold incident on a potassium metal plate. The frequency of potassium. photo electrons emitted from the W = hfo potassium plate were collected by a fo = W/h cathode and anode at varying voltages to = 2.3/4.14 x 10-15 obtain the curve in Figure 3. Potassium = 5.56 x 1014 Hz has a work function of 2.3 eV. Q18: Which of the following (A-D) would occur if the frequency was decreased to less than the threshold frequency? A Increased photocurrent B Decreased photocurrent Q17: UV light of 200 nm was now shone onto the C Lower stopping potassium plate at the same intensity striking the potential cathode. Sketch the resulting curve on the graph in D No signal. Figure 3 above.
  • 35. 3.5 Electron Energy When initially placed into the field, the Before proceeding with more on electron will possess Electrical Potential the Photoelectric Effect a short Energy given by the product of the detour into the world of electron charge on the electron (q) and the size of energy is required. the voltage difference or accelerating When an electron is placed in a voltage (V) So, E.P.E. = qV region where a voltage difference At this point the electron is stationary exists (ie. an electric field), it will so its KE =0 undergo an energy conversion in passing through that voltage difference. The electron arrives at Plate 2 KE KE = 0 KE KE KE where all its initial E.P.E. has e- e- e- e- e- been converted to K.E. Thus EPE EPE EPE EPE E.P.E.at start = K.E.at finish EPE =0 Mathematically: qV = ½mv2 Plate 1, Plate 2, V=0 V = +V
  • 36. 3.6 Energy Units • Electrons are tiny and the amount of energy they carry, even when exposed to accelerating voltages of tens of K e- E millions of volts, is also tiny. e- • In order to deal with this situation the normal energy unit of Joules (a very large unit) can be replaced by a EPE special energy unit called the “electron-Volt” (eV). • Plate 1, V = 0 Plate 2, V = +1V The electron-Volt is defined as the energy change experienced by an electron in passing through a Voltage On Plate 1 at Plate 2 thethe e-of At arrival the E.P.E. of K.E. of 1 Volt. = qVe- hasx 10-19 J = 1 eV eV the = 1.6 increased by 1 • Mathematically: E.P.E. = qV Thus an electron passing through = (1.6 x 10 )(1) -19 1 thousand volts will have = 1.6 x 10-19 J increased its energy by 1keV and Thus 1 eV = 1.6 x 10-19 J an electron passing through 10 million volts will have increased its energy by 10 MeV
  • 37. 3.7 The Photoelectric Effect (6) • The ENERGY of the incoming photons depends upon the The Energy carried by the incoming FREQUENCY OF THE LIGHT. So, Photon is TOTALLY ABSORBED by blue photons are more energetic the electron with which it collides. than red ones. • Mathematically: Some of the photon energy is used free E = hf the electron from the metal lattice and the where E = Energy (J or eV) rest is converted into the Kinetic Energy h = Planck’s Constant of the ejected electron. This can be f = Frequency (Hz) summarised mathematically as: • Planck’s constant can have 2 values depending upon the energy units KEMAX = hf - W used: If Joules: h = 6.63 x 10-34 J s; where: if eV: h = 4.14 x 10-15 eVs KEMAX = Maximum KE of ejected electron (J or eV) hf = Photon Energy (J or eV) W= Work Function (J or eV) The term, W, called the Work Function, is the energy needed to bring the electron from within the metal lattice to the surface before it can be ejected.
  • 38. 3.8 The Photoelectric Effect (7) A plot of the KEMAX 1. Remember KE MAX = hf - W, 3. Different metals have of the ejected thus a graph of KE MAX vs different cut off electron versus the frequency is a straight line frequencies. frequency of the graph of the form y = mx + c 4. Different metals have incoming photons is with m = h and c = W different work functions. shown below. This 2. When KEMAX = 0, hf = W graph has a number and this frequency is called 5. All metals will produce of important the “cut off frequency” (fO) graphs with the same features: slope = h (Planck’s for that particular metal. constant) Light below this frequency, no matter how intense, will never liberate electrons from this K.E.MAX metal. (eV or J ) Slope = h Slope = h fo fo Frequency (Hz) W W
  • 39. Light & Matter Revision Question Type: Photoelectric effect Susan and Peter conducted a photo- electric experiment in which they used a light source and various filters to allow light of different frequencies to fall on the metal plate of a photo-electric cell. The maximum kinetic energy of any emitted photo-electrons was determined by Figure 3 shows the stopping measuring the voltage required, VS voltage, VS, as a function of the (stopping voltage), to just stop them reaching the collector electrode. The frequency (f) of the light falling on apparatus is shown in Figure 2. the plate. Table 1 shows the work functions for a series of metals.
  • 40. Light & Matter Revision Question Type: Photoelectric effect Q19: Use the information above to identify the metal surface used in Susan and Peter’s experiment. The work function corresponding to sodium could be found by drawing a line of best fit through the points on the graph to determine the y-intercept. Q20: Use the results in Figure 3 to calculate the value for Planck’s constant that Susan and Peter would have obtained from the data. You must show your working. The gradient of the line of best fit gave Planck’s constant as approximately 4.5 x 10-15 eV s.
  • 41. Light & Matter Revision Question Type: Photoelectric effect Measurements of the kinetic energy of electrons emitted from potassium metal were made at a number of frequencies. The results are shown in Figure 1. Q21: What is the minimum energy of a light photon that can eject an electron from potassium metal? 1.8 eV The work function for silver metal is higher than the work function for potassium metal. Q22: Which one of the graphs (A- D) would best describe the result if the experiment was repeated with silver metal instead of potassium metal?
  • 42. Light & Matter Revision Question Type: Photoelectric effect Some students are investigating the photoelectric effect. They shine light of different wavelengths onto a rubidium plate. They measure the maximum kinetic energy of photoelectrons emitted from the plate. Their data of maximum kinetic energy Q23: From the data on the graph, what is the of ejected photoelectrons minimum energy, W, required to remove as a function of the photoelectrons from the rubidium plate? frequency of incident light 2.0 eV is shown in Figure 1. The students shine light of wavelength λ = 400 nm In answering the following onto the rubidium plate. questions, you must use Q24: From the graph, with what maximum kinetic the data from the graph. energy would the photoelectrons be emitted? Take the speed of light to 1.0 eV be 3.0 × 108 m s-1
  • 43. 3.9 The Photoelectric Effect (8) PHOTOELECTRIC EFFECTS OBSERVED WAVE MODEL PREDICTIONS 1. Below a certain frequency, 1. The energy of the light beam different for different metals, NO arrives uniformly and continuously at photoelectrons are observed, no the metal surface. If the intensity matter how INTENSE the light. increases, the energy available to the electrons increases, so KEMAX of the ejected electrons should also increase. 2. The KEmax of the ejected 2. The energy of the light beam is photoelectrons is INDEPENDENT completely described by its INTENSITY, of the INTENSITY of the incident so the KEMAX of the ejected light, but DEPENDENT on the photoelectrons should be INDEPENDENT FREQUENCY of that light. of Frequency. 3. Because of the nature of the energy 3. The photoelectrons are emitted delivery process, electrons should take IMMEDIATELY the metal is exposed to SOME TIME to build enough energy to light above the cut off frequency. be ejected. A TIME DELAY should exist. These anomalies between the observed and predicted results meant the death of the wave theory as a complete explanation for light behaviour. A new theory was required.
  • 44. Light & Matter Revision Question Type: Photoelectric effect The students use a light source that Q25: Comment whether this emits a large range of frequencies. experimental evidence supports They use filters which allow only the wave like or the particle-like certain frequencies from the source theory of light. to shine onto the plate. Most of the students filters produce frequencies The experimental evidence supported below the cut-off frequency. the particle theory. Alice says that if they increase the The wave theory predicts that intensity of light, these frequencies photoelectrons would be emitted at below the cut-off any frequency if the intensity was frequency will be able to produce sufficient. emitted photoelectrons. They experiment and find Alice is the particle theory, the energy of the incorrect. photons is related to the frequency, not the intensity, so there would be no electrons emitted below the threshold frequency.
  • 45. 3.10 The Photoelectric Effect (9) As mentioned previously it Photons are neither Photons can be was in 1905 that Einstein waves nor particles, visualised as a series of proposed that light travels having properties individual particles each as a series of discrete units similar to particles when of which displays some or particles or “quanta” travelling through a wave like properties. which he called “photons” vacuum and when in a and each carried a discrete gravitational field, while amount of energy also having properties dependent upon the light’s similar to waves when Individual Photon frequency. (ie E = hf). refracting and interfering. Light beam Light Beams can be regarded as a series of photons radiating out from the source.
  • 46. 3.11 Practical Applications The photoelectric effect has practical applications in many areas: Solar Cells Spacecraft The photoelectric effect will cause spacecraft exposed to sunlight to develop a positive charge. This can get up to the tens of volts. This can be a major problem, as Night Vision Goggles other parts of the spacecraft in shadow develop a negative charge (up to several kilovolts) from nearby plasma, and the imbalance can discharge through delicate electrical components.
  • 47. Chapter 4 Topics covered: • Investigating the Electron • Electrons and Matter as Waves. • Electron Interference • Electron Diffraction • Electrons and X Rays • Photon Momentum. • Photons as Waves.
  • 48. 4.0 Investigating the Electron • Electrons are stripped from atoms and then used as individual particles in many applications. • One important application is in Cathode Ray Tubes - the basis for Cathode Ray Oscilloscopes • The electron is the smallest of the 3 (CRO’s), T.V. and non LCD video fundamental particles which make display units. up all atoms. • These devices rely on a stream of • It carries the basic unit of electric “energetic” electrons, which are charge (1.6 x 10-19 C) “boiled off” a hot wire filament. • It has a mass of 9.1 x 10-31 kg • Their energy is then increased by • When in an atom, the electron being accelerated through a large circulates around the nucleus BUT Voltage. only in certain allowed positions or • Their trajectory can be manipulated orbits. by using magnetic or electric fields. • This restriction on position means that electrons are behaving more like waves than the particles we know them to be. • It was not until the development of “Quantum Mechanics” that the problem of restricted position was satisfactorily explained.
  • 49. 4.1 Electrons and Matter as Waves In 1924 French Physicist Louis DeBroglie suggested that “the universe is symmetrical” so, if radiation (light) A racing car, mass 800 kg, travelling at displayed both particle and wave 200 kmh-1 (55.6 ms-1) has a DeBroglie like properties simultaneously, wavelength of: matter should also display similar λ = h/p = h/mv = 6.63 x 10-34/(800)(55.6) behaviours. = 1.5 x 10-38 m • DeBroglie suggested that matter (with An indescribably small number. Too mass, m, moving with velocity, v), will small to measure or even notice. have a wavelength (called the DeBroglie wavelength) associated with its motion. v • This wavelength is calculated from the momentum equation: An electron of mass 9.1 x 10-31 kg travelling λ = h/p = h/mv at 1.4 x 104 ms-1 has a DeBroglie wavelength where λ = DeBroglie wavelength (m) of: h = Planck’s Constant λ = h/p = h/mv p = Particle Momentum = 6.63 x 10-34/(9.1 x 10-31)(1.4 x 104) (kgms ) -1 m = Mass (kg) = 5.2 x 10-8 m v = Velocity (ms ) -1 A measurable quantity, so electrons are able to display wave like properties like interference and diffraction.
  • 50. Light & Matter Revision Question Type: De Broglie Wavelength A sketch of a cathode ray tube (CRT) is Q26: Calculate the de Broglie shown in Figure 5. In this device, electrons wavelength of the electrons. You of mass 9.10 × 10-31 kg are must show your working. accelerated to a velocity of 2.0 × 10 m s . A 7 -1 λ= h/mv fine wire mesh in which the gap between the wires is w = 0.50 mm the de Broglie wavelength was has been placed in the path of the electrons, found to be 3.64 x 10-11 m. and the pattern produced is observed on the Q27: Explain, with reasons, fluorescent screen. whether or not the students would observe an electron diffraction pattern on the fluorescent screen due to the presence of the mesh. For diffraction, the gap width must be the same order of magnitude as the wavelength. Since the wavelength was much smaller than the gap, no diffraction pattern would be observed.
  • 51. Light & Matter Revision Question Type: De Broglie Wavelength Neutrons are subatomic particles and, like electrons, can exhibit both particle-like and wave-like behaviour. A nuclear reactor can be used to produce a beam of neutrons, which can then be used in experiments. The neutron has a mass of 1.67 × 10-27 kg. The neutrons have a de Broglie wavelength of 2.0 × 10-10 m. Q28: Calculate the speed of the neutrons. λ= h/mv gives 2.0 × 103 ms–1 (or 1985 ms–1) Q29: The neutron beam is projected onto a metal crystal with interatomic spacing of 3.0 × 10-10 m. Would you expect to observe a diffraction pattern? Explain your answer. There would be a diffraction pattern because the wavelength is of the same order of magnitude as the interatomic spacing.
  • 52. 3.11 Practical Applications The photoelectric effect has practical applications in many areas: Solar Cells Spacecraft The photoelectric effect will cause spacecraft exposed to sunlight to develop a positive charge. This can get up to the tens of volts. This can be a major problem, as Night Vision Goggles other parts of the spacecraft in shadow develop a negative charge (up to several kilovolts) from nearby plasma, and the imbalance can discharge through delicate electrical components.
  • 53. 4.2 Electron Interference The now familiar Young’s Initially, with only small . . .. . double slit experiment is numbers, electrons arrive performed with the light at the screen in a random source replaced by an fashion. “electron gun” which fires Even as the numbers reach a single electron at a time. the thousands still nothing unusual appears on the screen. However as the numbers climb past 5000 the familiar light and dark bands begin to appear The total exposure time 8 electrons 270 electrons from picture (a) to the picture (d) was 20 min. This result shows particles (in this case electrons) can display wave like properties. 2000 electrons 6000. electrons
  • 54. 4.3 Electron Diffraction Just as waves diffract when hitting a solid object so electrons can diffract from a well-ordered arrangement of atoms on the surface of a sample. Electrons which have been accelerated through a potential of 30 to 500 volts (i.e., have The arrangement of the K.E.’s of 30 to 500 eV), have a spots is interpreted to Platinum Sample de Broglie wavelength between provide information about 2.2 x 10-10 m and 0.5 x 10-10 m. the ordered arrangement of This fits nicely into the range of atoms on the surface and distances between atoms in the distances between the solids and can therefore spots gives information on strongly diffract from them. the distance between the atoms. e- energy = 65 eV
  • 55. Light & Matter Revision Question Type: Electron Diffraction A beam of electrons, pass through two very thin slits (approximately 10-10 m) Q30: Explain whether the and are detected on a electron detector particle model or the wave screen as shown in Figure 5. model best explains the expected observations from this experiment. The wave model best explains this. The wave model is better: Interference pattern (a wave phenomenon) will be observed. The Particle model would not predict the interference pattern, rather two zones where the electrons would strike the screen.
  • 56. 4.4 Electrons and X Rays Diffraction Pattern Al foil High energy X Ray Beam electron beam A high energy electron beam is fired at an The distance between the bright lines in Aluminium target both patterns is the same, meaning the The diffraction pattern wavelengths of both beams was the same. shown is produced. Hence if λX Ray is known, then the De When the electron beam is Broglie wavelength of the electrons is replaced with an X Ray also known. beam a somewhat similar diffraction is produced
  • 57. Light & Matter Revision Question Type: X ray Energy A beam of X-rays, wavelength λ = 250 pm (250 × 10-12 m), is directed onto a thin aluminium foil as shown in Figure 4a. The X-rays scatter from the foil onto the photographic film. Q31 : Calculate the energy, in keV, of these X-rays. E=hc/λ = (4.14 x 10-15)(3.0 x 108)/ 250 x 10-12) = 4.97 x 103 ev = 4.97 keV or 5.0 keV
  • 58. Light & Matter Revision Question Type: X ray Diffraction After the X-rays pass through the foil, Q32: Explain why the electrons a diffraction pattern is formed as produce a diffraction pattern similar shown in Figure 4b. to that of the X-rays. In a later experiment, the X-rays are Electrons have a de Broglie replaced with a beam of energetic wavelength which was the same electrons. Again, a diffraction pattern (or very similar) to the wavelength is observed which is very similar to of the X-rays. the X-ray diffraction pattern. This is Q33: Assuming the two diffraction shown in Figure 4c. patterns are identical, estimate the momentum of the electrons. Include the unit. p=h/λ = (6.63 x 10-34)/(250 x 10-12) = 2.7 x 10-24 kgms-1 The diffraction pattern were the same, so the wavelength of the electrons must equal that of the X- rays.
  • 59. Light & Matter Revision Question Type: X ray & electron Diffraction In 1927, G.I. Thomson fired a beam of electrons through a very thin metal foil and the emerging electrons formed circular patterns, similar to the pattern obtained when the exercise was repeated with a beam of X-rays. The following diagram shows a comparison of the patterns obtained. Q35: If the X-rays have frequency of Q34: How does this evidence support the 1.5 x 1019 Hz, what is the wavelength concept of matter waves? of the electrons? The circular pattern is a diffraction pattern. With X-rays, the bright lines λ = c/f = 3.0 x 108/1.5 x 1019 represent regions of constructive = 2.0 x 10-11 m interference alternating with regions of Q36: What is the momentum of an X- destructive interference. ray photon? The corresponding electron pattern shows similar diffraction properties p = h/λ = 6.63 x 10-34/2.0 x 10-11 which suggests electrons have wave = 3.3 x 10-23 kgms-1 properties, as diffraction is a wave phenomenon.
  • 60. 4.5 Evidence for Photons as Waves It appears trying to predict the fate of individual photons is not possible, Young’s double slit experiment is but as their numbers build the wave well known as evidence for the wave model allows the prediction of their like nature of light. average behaviour with great This experiment creates difficulties accuracy. for the particle (photon) model. Thus photons (en masse) can display How can individual photons, which wave like properties although they must pass through one or other of are generally regarded as discrete, the slits, interact with themselves to individual particles. produce an interference pattern ? When the experiment is carried out at extremely low light intensities, it is found that, for a while, no interference pattern is noticed. However, as time passes the number of photons arriving at the screen builds and an interference pattern does emerge with more photons going to the areas of bright bands and less to areas of dark bands.
  • 61. 4.6 Photon Momentum • When a light beam strikes a surface, the individual photons will transfer MOMENTUM as they are absorbed or reflected and so will exert a pressure on that surface. • The size of the MOMENTUM can be calculated from p = E/c …………(1) where, p = Photon Momentum (kgms-1) E = Photon Energy (J or eV) c = Speed of Light (ms-1) • Using Photon Energy, E = hf, Equation 1 can be rewritten as: p = hf/c …………(2) • Using the Wave Equation, c = fλ • Equation 2 can be rewritten as: p = hf/f λ p = h/λ
  • 62. Light & Matter Revision Question Type: Photon energy A monochromatic violet light of wavelength 390 nm is emitted by a light source with a power of 200 Watt. Q37: How many photons leave this light source in a 10 second interval? P = W/t = E/t  E = Pt = (200)(10) = 2000 J Each photon has Energy = hc/λ = = (6.63 x 10-34)(3.0 x 108)/(390 x 10-9) = 5.1 x 10-19 J No of photons in 2000 J = 2000/(5.1 x 10-19) = 3.92 x 1021 Q38: What is the momentum of each of these photons? p = h/λ = 1.7 x 10-27 kgms-1
  • 63. Chapter 5 • Topics covered: • Quantised Energy Levels for Atoms. • Emission Spectra • Absorption Spectra • Solar Spectrum. • Quantised Energy Levels • Wave Particle Duality.
  • 64. 5.0 Quantised Energy Levels for Atoms • The Bohr model for the atom, restricting electrons to The 1st five certain regions or orbits with electron shells fixed energy levels, worked with their standing well in describing atomic structure and behaviour. waves • But it was not until DeBroglie suggested electrons had a wavelength, was there a satisfactory explanation of why fixed orbits and energies existed. • DeBroglie suggested that for Disallowed orbit an electron to survive in an orbit, it must form a standing wave which represents a stable state for the electron. • Only certain radius orbits allow the standing wave to be set up, so electrons can only exist certain regions. Allowed Orbit
  • 65. 5.1 Emission Line Spectra When atoms of substances are exposed to The energy absorbed corresponds certain kinds of energy sources, the to jumps between the fixed energy electrons surrounding the nucleus can levels of the electron shells within gain energy and move up to higher energy the atom levels, called “excited states”. The excited electrons return to their ground state by emitting a photon of a certain frequency (colour) The emitted photons from all electrons transitions form “a picture” called an “Emission Spectrum”. These Spectra are unique to each element and are used to identify unknowns in a field called spectroscopy. Helium Emission Spectra Sodium Emission Spectra
  • 66. Light & Matter Revision Question Type: Emission Spectra A class looks at spectra from two sources. Q40: State the electron mechanism, i. an incandescent light globe in each of the sources below, that ii. a mercury vapour lamp produces each spectrum. They observe that the spectra are i. Incandescent light globe of different types. ii. Mercury vapour lamp Q39: State the type of spectrum i Thermal motion of free electrons seen from each source. i. Incandescent light globe ii Energy transition of bound electrons ii. Mercury vapour lamp i Continuous (or broad) ii Discrete (or individual lines)
  • 67. 5.2 Absorption Line Spectra If you look at the spectrum from a common incandescent light bulb, you'll see a relatively smooth spectrum with no part being brighter or darker than any other part. This type of a spectrum is called a continuous spectrum. Electrons in an atom can jump between The pattern of absorption lines in discreet energy levels by absorbing a a spectrum will be unique to a photon. chemical element so we can use So certain atoms under certain absorption lines to detect the conditions will absorb very specific presence of specific elements in wavelengths of light dependent on the astronomical objects. configuration of their electrons. This will remove energy and leave blank An emission spectrum is simply the spaces (dark lines) in the spectrum. reverse of an absorption spectrum.
  • 68. Light & Matter Revision Question Type: Absorption Spectra Figure 5a shows part of the emission spectrum of hydrogen in more detail. With a spectroscope, Val examines the spectrum of light from the sun. The spectrum is continuous, with colours ranging from red to violet. However there were black lines in the spectrum, as shown in Figure 5b. Q41: Explain why these dark lines are present in the spectrum from the sun. The dark lines exist because photons are absorbed which correspond to the energy levels in hydrogen. This indicates the presence of hydrogen.
  • 69. 5.3 The Solar Spectrum Solar Spectrum When the “White Light” arriving at the Earth’s surface is passed through a prism, it is broken up into its constituent colours. This produces the “solar spectrum” It was later discovered that each In 1802 English chemist William Wollaston was chemical element was the first to note the appearance of a number of associated with a set of spectral dark lines in the solar spectrum. lines, and that the dark lines in the solar spectrum were caused In 1814, Joseph Von Fraunhofer by absorption by those elements independently rediscovered the in the upper layers of the sun. lines and began a systematic Some of the observed features study and careful measurement of are also caused by absorption in the wavelength of these features. oxygen molecules in Earth’s In all, he mapped over 570 lines. atmosphere.
  • 70. 5.4 Quantised Energy Levels The energy is associated with Electrons surrounding the nucleus these “jumps” is given by E = hf can “jump” up and down between where allowed energy levels by absorbing E = Energy of the emitted photon or emitting a photon. for a downward jump OR the The ABSORPTION and EMISSION energy absorbed from an incident SPECTRA for atoms show the photon for an upward jump. (eV) energy values associated with the h = Planck’s Constant “jumps”. f = Frequency of Photon (Hz) Mercury Energy Levels Ionisation 10.4 eV n=5 8.8 eV n=4 6.7 eV n=3 Electronsabsorbing State5(n = 3 can of States in = 2 to n = represent The n excited state n 10.4 Electrons Ground more than = 1) eV 4.9 eV n=2 energy from a photon1.8 eV by: be represents thea state electron 2. By emitting lowest photon return to the groundcollision willfor allowed energy levels stripped completely4.9 eV photon Emitting a single 6. 7the Mercury electrons a from eV atoms. 1. followed byin Mercuryphoton energy level atom leaving it ionised OR 0 eV n=1 Ground State
  • 71. Light & Matter Revision Question Type: Energy Levels The spectrum of photons emitted by Q42: What is the lowest energy excited atoms is being investigated. photon that could be emitted Shown in Figure 6 is the atomic energy from the excited atoms? level diagram of the particular atom The smallest energy gap was being studied. Although most of the that from n = 3 to n = 2, an atoms are in the ground state, some energy difference of 1.8 eV. atoms are known to be in n = 2 and n = Q43: Calculate the wavelength of 3 excited states. the photon emitted when the atom changes from the n = 2 state to the ground state (n = 1). Data: h = 4.14 × 10-15 eV s , c = 3.0 × 108 m s-1 The energy of the photon was 3.4 eV. By substituting this into the equation E =hc/λ, the wavelength was 3.65 x 10-7 m.
  • 72. Light & Matter Revision Question Type: Energy Levels Figure 2 shows the energy levels of a sodium atom. A sodium atom is initially in an n = 4 excited state. Q44: Calculate the highest frequency of light that this sodium atom could emit. ΔE = 3.61 = h f Hence f = 8.7 x 1014 Hz Figure 2 shows that electrons in a Electrons have an equivalent or de sodium atom can only occupy specific Broglie wavelength. energy levels. Q45: Describe how the wave nature of Only orbits of an integral number of electrons can explain this. wavelengths are allowed as these will form a standing wave
  • 73. Light & Matter Revision Question Type: Energy Level Diagrams Part of the visible region of the spectrum of light emitted from excited hydrogen gas has three lines as shown in Figure 3. The energy level diagram for the hydrogen atom is shown in Figure 4. The binding energy is 13.6 eV. Q46: What is the energy of the photons with a wavelength 434.1 nm in Figure 3? E = hc/λ = 2.86 eV Q47: A different photon has an energy of 3.0 eV. On Figure 4 indicate with an arrow the electron transition that leads to emission of a photon of light with this energy.
  • 74. 5.5 The Wave - Particle Duality • As you can see from what has been presented, attempting to characterise light as only wave like or only particle like in nature has failed. • In fact, as of today, light (and matter) are regarded as some part particle like and some part wave like in nature. • This conflict is summarised in the use of the term “wave-particle duality” to describe light’s known BUT THIS IS WHAT PHYSICS IS ALL behaviour. ABOUT – • This is an unsatisfactory situation, PUSHING THE ENVELOPE – as it defies our need for simple all EXPLORING THE UNKNOWN – encompassing explanations for the TRYING TO FIT RATIONAL AND LOGICAL behaviours we observe around us and in the universe. EXPLANATIONS TO THAT WHICH HAS BEEN OBSERVED. • It would appear, at this stage, we have not yet unravelled all the details of the behaviour of light and matter.